t04 - risks & cost of capital

96
MANAGEMENT ADVISORY SERVICES RISKS INVESTMENT RISK 1 . Catherine & Co. has extra cash at the end of the year and is analyzing the best way to invest the funds. The company should invest in a project only if A. The expected return on the project exceeds the return on investments of comparable risk. B. The return on investments of comparable risk exceeds the expected return on the project. C. The expected return on the project is equal to the return on investments of comparable risk. Gleim D. The return on investments of comparable risk equals the expected return on the project. Interest-rate Risk 2 . Which of the following are components of interest-rate risk? Gleim A. Purchasing-power risk and default risk. C. Portfolio risk and reinvestment-rate risk. B. Price risk and market risk. D. Price risk and reinvestment-rate risk. 4. Long-term government bonds have: A . Interest rate risk C. Market risk B. Default risk D. None of the above B & M Purchasing-power Risk Default Risk 3 . The marketable securities with the least amount of default risk are (E) a. Federal government agency securities. c. Repurchase agreements. b . U.S. Treasury securities. d. Commercial paper. CMA 0691 1-11 26. The portion of the risk that can be eliminated by diversification is called: A. Unique risk C. Interest rate risk B. Market risk D . Default risk B & M Market Risk 4 . The type of risk that is not diversifiable and even affects the value of a portfolio is (E) A. Purchasing-power risk. C. Nonmarket risk. B. Market risk. D. Interest-rate risk. Gleim Nonmarket Risk or Company-specific Risk 27. The unique risk is also called the: A. Unsystematic risk C. Firm specific risk B. Diversifiable risk D . All of the above B & M 29. As the number of stocks in a portfolio is increased: A . Unique risk decreases and approaches to zero B. Market risk decrease C. Unique risk decreases and becomes equal to market risk B & M 5 . In capital market analysis, the nonsystematic risk RPCPA, AICPA. CMA & CIA EXAMINATION QUESTIONS Page 1 of 96

Upload: elsa-mendoza

Post on 19-May-2017

270 views

Category:

Documents


19 download

TRANSCRIPT

Page 1: T04 - Risks & Cost of Capital

MANAGEMENT ADVISORY SERVICES RISKS

INVESTMENT RISK1. Catherine & Co. has extra cash at the end of the year and is analyzing the best way to invest

the funds. The company should invest in a project only if A. The expected return on the project exceeds the return on investments of comparable risk. B. The return on investments of comparable risk exceeds the expected return on the project. C. The expected return on the project is equal to the return on investments of comparable

risk. GleimD. The return on investments of comparable risk equals the expected return on the project.

Interest-rate Risk 2. Which of the following are components of interest-rate risk? Gleim

A. Purchasing-power risk and default risk. C. Portfolio risk and reinvestment-rate risk.B. Price risk and market risk. D. Price risk and reinvestment-rate risk.

4. Long-term government bonds have: A. Interest rate risk C. Market riskB. Default risk D. None of the above B & M

Purchasing-power Risk Default Risk 3. The marketable securities with the least amount of default risk are (E)

a. Federal government agency securities. c. Repurchase agreements.b. U.S. Treasury securities. d. Commercial paper. CMA 0691 1-11

26. The portion of the risk that can be eliminated by diversification is called: A. Unique risk C. Interest rate riskB. Market risk D. Default risk B & M

Market Risk4. The type of risk that is not diversifiable and even affects the value of a portfolio is (E)

A. Purchasing-power risk. C. Nonmarket risk.B. Market risk. D. Interest-rate risk. Gleim

Nonmarket Risk or Company-specific Risk27. The unique risk is also called the:

A. Unsystematic risk C. Firm specific riskB. Diversifiable risk D. All of the above B & M

29. As the number of stocks in a portfolio is increased: A. Unique risk decreases and approaches to zero B. Market risk decrease

C. Unique risk decreases and becomes equal to market risk B & M

5. In capital market analysis, the nonsystematic riska. Is correlated with qualitative aspects of the underlying entity.b. Is correlated with quantitative aspects of the underlying entity.c. Cannot easily be overcome by individual investors.d. Is considered random. Gleim

6. Which of the following statements is correct? (E)

1 . Answer (A) is correct. Investment risk is analyzed in terms of the probability that the actual return on an investment will be lower than the expected return. Comparing a project's expected return to the return on an asset of similar risk, helps determine whether the project is worth investing in. If the expected return on a project exceeds the return on an asset of comparable risk, then the project should be pursued. Answer (B) is incorrect because a project should be pursued only if its expected return exceeds the return on investments of similar risk. Answer (C) is incorrect because a project should be pursued only if its expected return exceeds the return on investments of similar risk. Answer (D) is incorrect because a project should be pursued only if its expected return exceeds the return on investments of similar risk.

2 . Answer (D) is correct. Interest-rate risk is the risk of fluctuations in the value of an asset due to changes in interest rates. One component of interest-rate risk is price risk, which is portrayed as a decline in the value of bonds as interest rates increase. Reinvestment-rate risk is another component of interest-rate risk. If interest rates decline, lower returns will be available for reinvestment of interest and principal payments received. Answer (A) is incorrect because purchasing-power risk concerns inflation, and default risk concerns nonpayment by the debtor. Answer (B) is incorrect because market risk concerns price changes in the overall securities markets. Answer (C) is incorrect because portfolio risk is the risk remaining in a portfolio after diversifying investments.

3 . REQUIRED: The marketable securities with the least default risk.DISCUSSION: (B) The marketable securities with the lowest default risk are those issued by the federal government because they are backed by the full faith and credit of the U.S. government an are therefore the least risky form of investment.Answer (A) is incorrect because securities issued by a federal agency are first backed by that agency and secondarily by the U.S. government. Agency securities are issued by agencies and corporations created by the federal government, such as the Federal Housing Administration. Answer (C) is incorrect because repurchase agreements could become worthless if the organization agreeing to make the repurchase goes bankrupt. Answer (D) is

RPCPA, AICPA. CMA & CIA EXAMINATION QUESTIONS Page 1 of 55

Page 2: T04 - Risks & Cost of Capital

MANAGEMENT ADVISORY SERVICES RISKS

a. Well diversified stockholders do not consider corporate risk when determining required rates of return.

b. Undiversified stockholders, including the owners of small businesses, are more concerned about corporate risk than market risk.

c. Empirical studies of the determinants of required rates of return (k) have found that only market risk affects stock prices.

d. Market risk is important but does not have a direct effect on stock price because it only affects beta.

e. All of the statements above are correct. Brigham

7. In theory, the decision maker should view market risk as being of primary importance. However, within-firm, or corporate, risk is relevant to a firm’s(M)a. Well-diversified stockholders, because it may affect debt capacity and operating income.b. Management, because it affects job stability.c. Creditors, because it affects the firm’s credit worthiness.d. Statements a and c are correct.e. All of the statements above are correct. Brigham

Portfolio RiskTotal Risk

incorrect because commercial paper is unsecured.

4 . Answer (B) is correct. Prices of all stocks, even the value of portfolios, are correlated to some degree with broad swings in the stock market. Market risk is the risk that changes in a stock's price will result from changes in the stock market as a whole. Market risk is commonly referred to as nondiversifiable risk. Answer (A) is incorrect because purchasing-power risk is the risk that a general rise in the price level will reduce the quantity of goods that can be purchased with a fixed sum of money. Answer (C) is incorrect because nonmarket risk is the risk that is influenced by an individual firm's policies and decisions. Nonmarket risk is diversifiable because it is specific to each firm. Answer (D) is incorrect because interest-rate risk is the risk that the value of an asset will fluctuate due to changes in the interest rate.

5 . Answer (D) is correct. Nonsystematic risk is considered to be unique to a firm and therefore random. In capital market theory, nonsystematic risk can largely be avoided through diversified investments.Answers (A) and (B) are incorrect because nonsystematic risk cannot be correlated with any variable, whether qualitative or quantitative. Answer (C) is incorrect because may be reduced through diversification.

6. Corporate risk Answer: b Diff: E

8. The risk of a single asset is A. Nonmarket risk. C. Portfolio risk.B. Total risk. D. Market risk. Gleim

9. An asset with high risk will have a(n) A. Low expected return. C. Increasing expected return.B. Lower price than an asset with low risk. D. High standard deviation of returns. Gleim

10. Risk to a company is affected by both project variability and how project returns correlate with those of the company’s prevailing business. Overall company risk will be lowest when a project’s returns exhibit CIA 1186 IV-39a. Low variability and negative correlation. c. High variability and positive correlation.b. Low variability and positive correlation. d. High variability and no correlation.

Liquidity Risk 11. The risk that securities cannot be sold at a reasonable price on short notice is called

A. Default risk. C. Purchasing-power risk.B. Interest-rate risk. D. Liquidity risk. CIA 1190 IV-51

12. When purchasing temporary investments, which one of the following best describes the risk associated with the ability to sell the investment in a short period of time without significant price concessions? (E)A. Interest rate risk. C. Financial risk.B. Purchasing power risk. D. Liquidity risk. CMA 0697 1-11

Business Risk 13. Business risk is the risk inherent in a firm's operations that excludes financial risk. It depends

on all of the following factors except (E)A. Amount of financial leverage. C. Demand variability.B. Sales price variability. D. Input price variability. Gleim

14. Business risk excludes such factors as A. Financial risk. C. Demand variability.B. Amount of operating leverage. D. Fluctuations in suppliers' prices. Gleim

1. A decrease in the debt ratio will generally have no effect on __________ ______. (E)a. Financial risk.b. Total risk.c. Business risk.d. Market risk. Brigham

RPCPA, AICPA. CMA & CIA EXAMINATION QUESTIONS Page 2 of 55

Page 3: T04 - Risks & Cost of Capital

MANAGEMENT ADVISORY SERVICES RISKS

2. Business risk is concerned with the operations of the firm. Which of the following is not associated with (or not a part of) business risk? (E)a. Demand variability.b. Sales price variability.c. The extent to which operating costs are fixed.d. Changes in required returns due to financing decisions.e. The ability to change prices as costs change. Brigham

15. Which of the following affects a firm’s business risk? (E)a. The level of uncertainty about future sales.b. The degree of operating leverage.c. The degree of financial leverage.d. Statements a and b are correct.e. All of the statements above are correct. Brigham

Financial Risk *. Which of the following would increase risk? (M)

a. Increase the level of working capital.b. Change the composition of working capital to include more liquid assets.c. Increase the amount of short-term borrowing.d. Increase the amount of equity financing. RPCPA 1091

16. A firm’s financial risk is a function of how it manages and maintains its debt. Which one of the following sets of ratios characterizes the firm with the greatest amount of financial risk?A. High debt-to-equity ratio, high interest coverage ratio, stable return on equity. B. Low debt-to-equity ratio, low interest coverage ratio, volatile return on equity. C. High debt-to-equity ratio, low interest coverage ratio, volatile return on equity. D. Low debt-to-equity ratio, high interest coverage ratio, stable return on equity. CMA 1291 1-

4

Business and financial risk3. Which of the following statements is most correct? (E)

a. A firm’s business risk is solely determined by the financial characteristics of its industry.b. The factors that affect a firm’s business risk are determined partly by industry

characteristics and partly by economic conditions. Unfortunately, these and other factors that affect a firm’s business risk are not subject to any degree of managerial control.

c. One of the benefits to a firm of being at or near its target capital structure is that financial flexibility becomes much less important.

d. The firm’s financial risk may have both market risk and diversifiable risk components.Brigham

Exchange-rate Risk 17. The risk of loss because of fluctuations in the relative value of foreign currencies is called

A. Expropriation risk. C. Multinational beta.B. Sovereign risk. D. Exchange rate risk. CIA 1191 IV-60

RPCPA, AICPA. CMA & CIA EXAMINATION QUESTIONS Page 3 of 55

Page 4: T04 - Risks & Cost of Capital

MANAGEMENT ADVISORY SERVICES RISKS

18. O & B Company, a U.S. corporation, is in possession of accounts receivable denominated in German deutsche marks. To what type of risk are they exposed? (E)A. Liquidity risk. C. Exchange-rate risk.B. Business risk. D. Price risk. Gleim

19. Bonner Electronics has subsidiaries in several international locations and is concerned about its exposure to foreign exchange risk. In countries where currency values are likely to fall, Bonner should encourage all of the following policies except A. Granting trade credit whenever possible. B. Investing excess cash in inventory or other real assets. C. Purchasing materials and supplies on a trade credit basis. CFM Sample Q. 5D. Borrowing local currency funds if an appropriate interest rate can be obtained.

20. A firm may seek to avoid exchange-rate risk by A. Maintaining a net monetary debtor position in countries with strengthening currencies. B. Maintaining a net monetary creditor position in countries with weakening currencies. C. Avoiding diversification of foreign-currency transactions. GleimD. Buying forward exchange contracts to cover liabilities denominated in a foreign currency.

Cultural Risk56. A U.S. manufacturer of which of the following goods would be likely to face the most cultural

risks in operating globally?a. Furniture c. Clothingb. Automobiles d. Food Barfields

57. A U.S. manufacturer of which of the following goods would be likely to face the fewest cultural risks in operating globally?a. Toys c. Clothingb. Food d. Furniture Barfields

Political Risk 58. Which of the following would be considered a political risk in doing business globally?

a. Asset expropriation c. Workplace diversityb. Inflation d. All of the above Barfields

21. Political risk may be reduced by A. Entering into a joint venture with another foreign company. B. Making foreign operations dependent on the domestic parent for technology, markets, and

supplies. C. Refusing to pay higher wages and higher taxes. D. Financing with capital from a foreign country. Gleim

Comprehensive*. All of the following statements are correct except:

a. The matching of asset and liability maturities is considered desirable because this strategy minimizes interest rate risk.

b. Default risk refers to the inability of the firm to pay off its maturing obligations.c. The matching of assets and liability maturities lowers default risk.d. An increase in the payables deferral period will lead to a reduction in the need to non-

spontaneous funding. RPCPA 1095

RPCPA, AICPA. CMA & CIA EXAMINATION QUESTIONS Page 4 of 55

Page 5: T04 - Risks & Cost of Capital

MANAGEMENT ADVISORY SERVICES RISKS

RISK MANAGEMENT METHODSPortfolio Theory1. Portfolio Theory was first developed by:

A. Merton Miller C. Harry MarkowitzB. Franco Modigliani D. Richard Breadey B & M

12. A portfolio will a usually contain: A. One riskless asset C. One risky assetB. Two or more assets D. None of the above B & M

Portfolio ManagementEfficient Portfolio 25. Efficient portfolios are those which offer:

A. Highest expected return for a given level of risk B. Highest risk for a given level of expected return C. The maximum risk and expected return D. All of the above B & M

33. Efficient portfolios are portfolios that: A. Offer the highest rate of return for a given level of risk B. Offer the lowest rate of return for a given level of risk C. Offer the lowest level of risk for a given rate of return D. A and C B & M

Feasible Portfolio22. A feasible portfolio that offers the highest expected return for a given risk or the least risk for a

given expected return is a(n) A. Optimal portfolio. C. Efficient portfolio.B. Desirable portfolio. D. Effective portfolio. Gleim

Optimal Portfolio23. An optimal portfolio of investments is (E)

A. Efficient because it offers the highest expected return. B. Any portfolio chosen from the efficient set of portfolios. C. Any portfolio chosen from the feasible set of portfolios. D. Tangent to the investor's highest indifference curve. Gleim

24. A company uses portfolio theory to develop its investment portfolio. If the company wishes to obtain optimal risk reduction through the portfolio effect, it should make its next investment in A. An investment that correlates negatively to the current portfolio holdings.

B. An investment that is uncorrelated to the current portfolio holdings. C. An investment that is highly correlated to the current portfolio holdings. D. An investment that is perfectly correlated to the current portfolio holdings. CIA 0591 IV-48

Minimum Variance Portfolio24. Florida Company (FC) and Minnesota Company (MC) are both service companies. Their

historical return for the past three years are: FC: -10%,15%, 25%; MC: 10%, 6%, 32%. Which portfolio is better? A. Portfolio with 50% in FC and 50% in MC C. Investment in FCB. Minimum variance portfolio D. None of the above B & M

26. Is the minimum variance portfolio an efficient portfolio? A. Yes C. Not necessarilyB. No B & M

Well-Diversified Portfolio30. In a well diversified portfolio, the type of risk remaining is:

A. Individual security risk C. Total riskB. Zero risk D. Market risk B & M

31. A well-diversified portfolio has negligible: A. Systematic risk C. Market riskB. Unique risk D. None of the above B & M

Portfolio Matrix Analysis25. Which one of the following planning techniques is most likely to be used to determine which

business units will receive additional capital and which will be divested? (D)A. Competitive strategies model. C. Scenario development.B. Portfolio matrix analysis. D. Situational analysis. CMA Samp Q3-9

Unsystematic Risk 26 In a well diversified portfolio (M) Bodie

a. market risk is negligible. c. unsystematic risk is negligible.b. systematic risk is negligible. d. nondiversifiable risk is negligible.

Systematic Risk27. Capital Asset Pricing Theory asserts that portfolio returns are best explained by: (E)

a. economic factors. c. systematic risk.b. specific risk. d. diversification. Bodie

RPCPA, AICPA. CMA & CIA EXAMINATION QUESTIONS Page 5 of 55

Page 6: T04 - Risks & Cost of Capital

MANAGEMENT ADVISORY SERVICES RISKS

Standard Deviation vs. Beta Coefficient23. Standard deviation and beta both measure risk, but they are different in that (E)

a. beta measures both systematic and unsystematic risk.b. beta measures only systematic risk while standard deviation is a measure of total isk.c. beta measures only unsystematic risk while standard deviation is a easure of total risk.d. beta measures both systematic and unsystematic risk while standard deviation measures

only systematic risk. Bodiee. beta measures total risk while standard deviation measures only nonsystematic risk.

Market Price of Risk28. The market price of risk (M)

a. is the risk premium divided by the standard deviation of the market returns.b. has a reward-to-risk ratio of [E(rM ) - rf]/2

M.c. is the price of a U. S. T-bill.d. a and b.e. a and c. Bodie

Risk Level of Securities29. Which of the following classes of securities are listed in order from lowest risk/opportunity for

return to highest risk/opportunity for return? (E)A. U.S. Treasury bonds; corporate first mortgage bonds; corporate income bonds; preferred

stock. B. Corporate income bonds; corporate mortgage bonds; convertible preferred stock;

subordinated debentures. C. Common stock; corporate first mortgage bonds; corporate second mortgage bonds;

corporate income bonds. CIA 0589 IV-49D. Preferred stock; common stock; corporate mortgage bonds; corporate debentures.

1. Which of the following portfolios have the least risk? A. A portfolio of treasury bills B. A portfolio of long term United States Government bonds C. Standard and Poor's composite index D. Portfolio of common stocks of small firms B & M

30. From the viewpoint of the investor, which of the following securities provides the least risk?a. Mortgage bond. c. Income bond.b. Subordinated debenture. d. Debentures. CIA 1191 IV-50

31. The expected rate of return for the stock of Corn Enterprises is 20%, with a standard deviation of 15%. The expected rate of return for the stock of Must Associates is 10%, with a standard deviation of 9%. The riskier stock is

A. Corn because its return is higher. B. Corn because its standard deviation is higher. C. Must because its standard deviation is higher. D. Must because its coefficient of variation is higher. CMA 0692 1-6

Risky Investment vs. Riskless Investment32. The difference between the required rate of return on a given risky investment and that on a

riskless investment with the same expected return is the A. Risk premium. C. Standard deviation.B. Coefficient of variation. D. Beta coefficient. CIA 1192 IV-48

Capital Asset Pricing Model33. The capital asset pricing model deals with risk and rates of return of a

A. Single security. B. Group of securities in a portfolio which follows a buy and hold strategy. C. Portfolio and how a new security affects that portfolio. D. Single fixed asset. CIA 0585 IV-28

34 According to the Capital Asset Pricing Model (CAPM) a well diversified portfolio's rate of return is a function of (E)a. market risk c. unique risk.b. unsystematic risk d. reinvestment risk. Bodie

Beta Coefficient37. The "beta" is a measure of:

A. Unique risk C. Total riskB. Market risk D. None of the above B & M

35. The level of risk that concerns investors who supply capital to a diversified company is A. Project risk (beta). B. Pure play risk (beta). C. The weighted average of project risk (betas). D. Accounting risk (beta). CIA 0592 IV-49

36. A company's beta value has decreased because of a change in its marketing strategy. Consequently, the discount rate applied to expected cash flows of potential projects will be A. Reduced. C. Unchanged.B. Increased. D. Zero. CIA 1185 IV-26

Portfolio Beta39. The beta of market portfolio is:

RPCPA, AICPA. CMA & CIA EXAMINATION QUESTIONS Page 6 of 55

Page 7: T04 - Risks & Cost of Capital

MANAGEMENT ADVISORY SERVICES RISKS

A. 0 C. +1.0B. +0.5 D. –1.0 B & M

40. The beta of a risk-free portfolio is: A. 0 C. +1.0B. +0.5 D. –1.0 B & M

34. Beta of Treasury bills portfolio is: A. Zero C. –1.0B. +0.5 D. +1.0 B & M

44. A stock with a beta of zero would be expected to: A. Have a rate of return equal to the risk-free rate B. Have a rate of return equal to the market risk premium C. Have a rate of return equal to zero D. Have a rate of return equal to the market rate of return B & M

Equity Beta37. If beta of debt is zero, then the beta of equity is equal to:

A. (1 + Debt-equity ratio)(beta of assets) C. (Beta of assets)/(debt-equity ratio)B. (Debt-equity ratio)(beta of assets) D. None of the above B & M

37. In many situations debt beta can be safely assumed to be zero. Under this assumption, equity beta can be expressed as: [E = market value of equity and D = market value of debt] A. equity beta = (1-(D/E.) (asset beta) C. equity beta = (asset beta)/(1+(D/E.)B. equity beta = (1+(D/E.) (asset beta) D. None of the above B & M

Asset Beta, Debt Beta & Equity Beta32. Which of the following is true?

A. bD > bA > bE C. bA > bE > bDB. bE > bA > bD D. None of the above are true B & M

34. Which of the following is true? A. bD < bA < bE C. bA < bE < bDB. bE < bA < bD D. None of the above are true B & M

Standard Deviation38. The variance or standard deviation is a measure of:

A. Total risk C. Market riskB. Unique risk D. None of the above B & M

2. Investments A and B both offer an expected rate of return of 12%. If the standard deviation of A is 20% and that of B is 30%, then investors would: A. Prefer A to B B. Prefer B to A C. Prefer a portfolio of A and B D. Cannot answer without knowing investor's risk preferences B & M

Coefficient of Variation24. Which of the following can be computed and compared for each alternative to determine the

relative riskiness of investments that have different levels of expected return?A. coefficient of variation C. standard deviationB. variance D. expected value Carter & Usry

37. The expected rate of return for the stock of Cornhusker Enterprises is 20%, with a standard deviation of 15%. The expected rate of return for the stock of Mustang Associates is 10% with a standard deviation of 9%. The riskier stock is (M)a. Cornhusker because the return is higher.b. Cornhusker because the standard deviation is higher.c. Mustang because the standard deviation is higher.d. Mustang because the return is lower.e. Mustang because the coefficient of variation is higher. CMA 0692 1-6

Variance27. In the formula for calculating the variance of an asset portfolio, how many are covariance

terms? A. [N(N-1)]/2 C. NB. N^2 D. None of the above B & M

28. In the formula for calculating the variance of an N-asset portfolio, how many are variance terms? A. [N(N-1)]/2 C. NB. N^2 D. None of the above B & M

32. The variance formula for a four stock portfolio contains: A. 4 individual variance terms and 6 unique covariance terms B. 3 individual variance terms and 6 unique covariance terms C. 6 individual variance terms and 9 unique covariance terms D. 6 individual variance terms and 6 unique covariance terms B & M

RPCPA, AICPA. CMA & CIA EXAMINATION QUESTIONS Page 7 of 55

Page 8: T04 - Risks & Cost of Capital

MANAGEMENT ADVISORY SERVICES RISKS

Covariance38. Which of the following specifically measures the volatility of returns together with their

correlation with the returns of other securities? (M)A. Variance. C. Coefficient of variation.B. Standard deviation. D. Covariance. Gleim

39. If the covariance of stock A with stock B is -.0076, then what is the covariance of stock B with stock A? A. +.0076 C. Greater than .0076.B. -.0076 D. Less than -.0076. Gleim

Indifference Curve40. An indifference curve represents combinations of portfolios having equal utility to the investor.

Given that risk and returns are plotted on the horizontal and vertical axes, respectively, and that the investor is risk averse, the curve has A. An increasingly steeper slope if the investor is less risk averse. B. A decreasingly negative slope if the investor's utility increases. C. An increasingly positive slope. D. A decreasingly positive slope. Gleim

Expected Rate of Return3. When stocks with the same expected return are combined into a portfolio, the expected return

of the portfolio is: A. Less than the average expected return value of the stocks B. Greater than the average expected return of the stocks C. Equal to the average expected return of the stocks D. Impossible to predict B & M

41. An investor uses the capital asset pricing model (CAPM) to evaluate the risk-return relationship on a portfolio of stocks held as an investment. Which of the following would not be used to estimate the portfolio's expected rate of return? (D)A. Expected risk premium on the portfolio of stocks. B. Interest rate for the safest possible investment. C. Expected rate of return on the market portfolio. D. Standard deviation of the market returns. CIA 1193 IV-47

Expected Return – Beta Relationship25. The expected return-beta relationship (M)

a. is the most familiar expression of the CAPM to practitioners.

b. refers to the way in which the covariance between the returns on a stock and returns on the market measures the contribution of the stock to the variance of the market portfolio, which is beta.

c. assumes that investors hold well-diversified portfoliosd. all of the above are true. Bodie

Comprehensive12 According to the Capital Asset Pricing Model (CAPM), which one of the following statements is

false? (M)a. The expected rate of return on a security decreases in direct proportion to a decrease n the

risk-free rate.b. The expected rate of return on a security increases directly with its beta.c. A fairly priced security has an alpha of zero.d. In equilibrium, all securities lie on the security market line. Bodie

Two-Stocks PortfolioCorrelation Coefficient36. For a two-stock portfolio, the maximum reduction in risk occurs when the correlation coefficient

between the two stocks is: A. +1 C. –0.5B. 0 D. –1 B & M

11. Maximum diversification is obtained by combining two stocks with a correlation coefficient equal to: A. +1.0 C. –1.0B. 0.0 D. +0.5 B & M

Correlation9. The correlation measures the:

A. Rate of movements of the return of individual stocks B. Direction of movement of the return of individual stocks C. Direction of movement between the returns of two stocks D. Stock market volatility B & M

42. The returns on two stocks can be correlated in values except those that are A. Positive. C. Neutral.B. Negative. D. Skewed. Gleim

Variance29. If the correlation between two stocks is +1, then a portfolio combining these two stocks will

have variance that is:

RPCPA, AICPA. CMA & CIA EXAMINATION QUESTIONS Page 8 of 55

Page 9: T04 - Risks & Cost of Capital

MANAGEMENT ADVISORY SERVICES RISKS

A. Less than the weighted average of the two individual variances B. More than the weighted average of the two individual variances C. Equal to the weighted average of the two individual variances D. None of the above B & M

Expected Return4. Stock A has an expected return of 20%, and stock B has an expected return of 12%. The risk

of Stock A as measured by the variance of the returns is twice that of stock B. If the two stocks are combined equally in a portfolio, what would be the expected return of the portfolio? A. 16% C. 20%B. 12% D. Need more information to answer B & M

Hedging43. When a firm finances each asset with a financial instrument of the same approximate maturity

as the life of the asset, it is applying A. Working capital management. C. Financial leverage.B. Return maximization. D. A hedging approach. CMA 1291 1-13

44. Contracts to hedge risk by exchanging cash flows include (E)

Gleim A. B. C. D.Interest-Rate Swaps Yes Yes No NoCurrency Swaps Yes No Yes No

45. The use of derivatives to either hedge or speculate results in A. Increased risk regardless of motive. B. Decreased risk regardless of motive. C. Offset risk when hedging and increased risk when speculating. D. Offset risk when speculating and increased risk when hedging. Gleim

46. A company has recently purchased some stock of a competitor as part of a long-term plan to acquire the competitor. However, it is somewhat concerned that the market price of this stock could decrease over the short run. The company could hedge against the possible decline in the stock's market price by CIA 0590 IV-57A. Purchasing a call option on that stock. C. Selling a put option on that stock.B. Purchasing a put option on that stock. D. Obtaining a warrant option on that stock.

Duration Hedging47. Duration hedging involves hedging interest-rate risk by matching the duration of assets with

the duration of liabilities. Which of the following is a true statement about duration hedging? (M)A. If duration increases, the volatility of the price of a debt instrument decreases. B. The goal of duration hedging is to equate the duration of assets with the duration of

liabilities. C. The firm is immunized against interest-rate risk when the total price change for assets

equals the total price change for liabilities. D. Duration is higher if the nominal rate on a debt instrument is higher. Gleim

Forward Contract48. A forward contract involves (E)

A. A commitment today to purchase a product on a specific future date at a price to be determined some time in the future.

B. A commitment today to purchase a product some time during the current day at its present price.

C. A commitment today to purchase a product on a specific future date at a price determined today.

D. A commitment today to purchase a product only when its price increases above its current exercise price. Gleim

RPCPA, AICPA. CMA & CIA EXAMINATION QUESTIONS Page 9 of 55

Page 10: T04 - Risks & Cost of Capital

MANAGEMENT ADVISORY SERVICES RISKS

49. If a corporation holds a forward contract for the delivery of U.S. Treasury bonds in 6 months and, during those 6 months, interest rates decline, at the end of the 6 months the value of the forward contract will have A. Decreased. B. Increased. C. Remained constant. D. Any of the answers may be correct, depending on the extent of the decline in interest

rates. Gleim

Futures Contract50. A distinguishing feature of a futures contract is that

A. Performance is delayed. C. Delivery is to be on a specific day. GleimB. It is a hedge, not a speculation. D. The price is marked to market each day.

51. An automobile company that uses the futures market to set the price of steel to protect a profit against price increases is an example of A. A short hedge. B. A long hedge. C. Selling futures to protect the company from loss. D. Selling futures to protect against price declines. Gleim

Interest rate futures contractInterest rate Swap52. In an interest rate swap, the first company

A. Sells its right to low interest rate financing at a financial institution to the second company that is seeking to borrow funds.

B. Agrees to service the debt of the second company by making interest payments directly to the bank of the second company, while the second company agrees in exchange to make interest payments to the bank of the first company.

C. Buys the outstanding public debt of the second company and swaps the interest payments it receives on that debt for the interest payments it must make on its own debt.

D. Agrees to exchange with the second company the difference between the interest charges on its own borrowings and the interest charges on the borrowings of the second company.

CIA 0596 IV-29

Interest Rate & Currency Swap53. Contracts to hedge risk by exchanging cash flows include

Gleim A. B. C. D.Interest-Rate Swaps Yes Yes No NoCurrency Swaps Yes No Yes No

METHODS OF ANALYZING RISKSensitivity Analysis54. Which of the following approaches would best analyze the risk of increasing the price of a

table by $50? (E) A. Sensitivity analysis. C. Informal method.B. Simulation analysis. D. Certainty equivalent adjustments. Gleim

Simulation Analysis55. Which method for measuring risk considers both the sensitivity of changing NPVs and the

range of values of the variables that are changed? A. Simulation analysis. C. Sensitivity analysis.B. The Capital Asset Pricing Model. D. Certainty equivalent adjustments. Gleim

Analysis of Pricing Technique50. The acronym APT stands for:

A. Arbitrage Pricing Model C. Analysis of Pricing TechniqueB. Asset Pricing Tool D. Analysis Pricing Theory B & M

51. A "factor" in APT is a variable that: A. Affects the return of risky assets in a systematic manner B. Correlates with risky asset returns in an unsystematic manner C. Is purely "noise" D. Affects the return of a risky asset in a random manner B & M

Three-factor Model55. The three factors in the Three-Factor Model are:

A. Market factor C. Book-to-market factorB. Size factor D. All of the above B & M

RISK-ADJUSTED DISCOUNT RATE56. Mega Inc., a large conglomerate with operating divisions in many industries, uses risk-adjusted

discount rates in evaluating capital investment decisions. Consider the following statements concerning Mega's use of risk-adjusted discount rates.I. Mega may accept some investments with internal rates of return less than Mega's overall

average cost of capital.II. Discount rates vary depending on the type of investment.III. Mega may reject some investments with internal rates of return greater than the cost of

capital.IV. Discount rates may vary depending on the division.Which of the above statements are correct? (D)A. I and III only. C. II, III, and IV only.

RPCPA, AICPA. CMA & CIA EXAMINATION QUESTIONS Page 10 of 55

Page 11: T04 - Risks & Cost of Capital

MANAGEMENT ADVISORY SERVICES RISKS

B. II and IV only. D. I, II, III, and IV. CMA Samp Q4-5

57. Dick Boe Enterprises, an all-equity firm, has a corporate beta coefficient of 1.5. The financial manager is evaluating a project with an expected return of 21 percent, before any risk adjustment. The risk-free rate is 10 percent, and the required rate of return on the market is 16 percent. The project being evaluated is riskier than Boe’s average project, in terms of both beta risk and total risk. Which of the following statements is most correct? (E)a. The project should be accepted since its expected return (before risk adjustment) is

greater than its required return.b. The project should be rejected since its expected return (before risk adjustment) is less

than its required return.c. The accept/reject decision depends on the risk-adjustment policy of the firm. If the firm’s

policy were to reduce a riskier-than-average project’s expected return by 1 percentage point, then the project should be accepted.d. Riskier-than-average projects should have their expected returns increased to reflect their added riskiness. Clearly, this would make the project acceptable regardless of the amount of the adjustment.

e. Projects should be evaluated on the basis of their total risk alone. Thus, there is insuffi-cient information in the problem to make an accept/reject decision. Brigham

58. Assume you are the director of capital budgeting for an all-equity firm. The firm’s current cost of equity is 16 percent; the risk-free rate is 10 percent; and the market risk premium is 5 percent. You are considering a new project that has 50 percent more beta risk than your firm’s assets currently have, that is, its beta is 50 percent larger than the firm’s existing beta. The expected return on the new project is 18 percent. Should the project be accepted if beta risk is the appropriate risk measure? Choose the correct statement. (M)a. Yes; its expected return is greater than the firm’s cost of capital.b. Yes; the project’s risk-adjusted required return is less than its expected return.c. No; a 50 percent increase in beta risk gives a risk-adjusted required return of 24 percent.d. No; the project’s risk-adjusted required return is 2 percentage points above its expected

return.e. No; the project’s risk-adjusted required return is 1 percentage point above its expected

return. Brigham

59. Assume you are the director of capital budgeting for an all-equity firm. The firm’s current cost of equity is 16 percent; the risk-free rate is 10 percent; and the market risk premium is 5 percent. You are considering a new project that has 50 percent more beta risk than your firm’s assets currently have, that is, its beta is 50 percent larger than the firm’s existing beta. The expected return on the new project is 18 percent. Should the project be accepted if beta risk is the appropriate risk measure? Choose the correct statement. (M)

a. Yes; its expected return is greater than the firm’s cost of capital.b. Yes; the project’s risk-adjusted required return is less than its expected return.c. No; a 50 percent increase in beta risk gives a risk-adjusted required return of 24 percent.d. No; the project’s risk-adjusted required return is 2 percentage points above its expected

return.e. No; the project’s risk-adjusted required return is 1 percentage point above its expected

return. Brigham

COST OF CAPITAL42. Cost of capital is (E)

a. The interest rate an entity must pay to borrow money.b. The return an entity’s stockholders expect on their investment .c. The rate of return the entity can earn from investing available cash. d. A concept of managerial finance incorporating all of the above. L & H

13. Cost of capital isa. The amount the company must pay for its plant assets.b. The dividends a company must pay on its equity securities.c. The cost the company must incur to obtain its capital resources.d. The cost the company is charged by investment bankers who handle the issuance of

equity or long-term debt securities. L & H

26. A dollar now is worth more than a dollar to be received in the future because ofa. Inflation. c. The opportunity cost of waiting.b. Uncertainty. d. None of the above. L & H

60. The theory underlying the cost of capital is primarily concerned with the cost of A. Long-term funds and old funds. B. Short-term funds and new funds. C. Long-term funds and new funds. D. Any combination of old or new, short-term or long-term funds. CMA 0692 1-13

61. Management knowledge of the cost of capital is useful for each of the following except (D)a. Making capital investment decisions.b. Managing working capital.c. Setting the maximum rate of return on new investments.d. Evaluating performance. Gleim

62. In referring to the graph of a firm's cost of capital, if e is the current position, which one of the following statements best explains the saucer or U-shaped curve?

RPCPA, AICPA. CMA & CIA EXAMINATION QUESTIONS Page 11 of 55

Page 12: T04 - Risks & Cost of Capital

MANAGEMENT ADVISORY SERVICES RISKS

Cost ofCapital(percent) e

Debt-to-Equity RatioA. The composition of debt and equity does not affect the firm's cost of capital. B. The cost of capital is almost always favorably influenced by increases in financial

leverage. C. The financial markets will penalize firms that borrow even in moderate amounts. D. Use of at least some debt financing will enhance the value of the firm. CMA 1288 1-5

26. The pre-tax cost of capital is higher than the after-tax cost of capital because (E)a. interest expense is deductible for tax purposes.b. principal payments on debt are deductible for tax purposes.c. the cost of capital is a deductible expense for tax purposes.d. dividend payments to stockholders are deductible for tax purposes. Barfield

63. The overall cost of capital is the A. Rate of return on assets that covers the costs associated with the funds employed. B. Average rate of return a firm earns on its assets. C. Minimum rate a firm must earn on high-risk projects. D. Cost of the firm's equity capital at which the market value of the firm will remain

unchanged. CMA 0692 1-11

*. Which of these statements are pertinent to cost of capital? (D)1. It is the expected return that investors demand for a given level of risk.2. It may be employed as a benchmark for the evaluation of performance.3. For investment decisions, it must be based on the current or prospective cost of the

various capital components rather than on their historical costs.4. It may also be used in acquisition analysis, liquidation studies and source of financing

decisions.5. It may differ from the hurdle rate used to reflect relative risk attributed to a specific project,

division, or business unit. RPCPA 1094a. All five statements. c. Statements 1, 2, 3, and 4 only.b. Statements 1, 2 and 3 only. d. Statements 1, 2, 4 and 5 only.

24. Which of the following is true?

a. Companies can raise common equity only by issuing new shares of common stock.b. There is no opportunity cost associated with use of retained earnings as a source of

common equity.c. Most large mature firms issue new shares of common stock on a regular basis.d. Companies can raise common equity by issuing new shares of common stock and

through retained earnings. S, S & S

25. Which of the following is incorrect?a. The after-tax cost of debt for a firm with losses is equal to the interest rate on the debt.b. Most debt is placed privately and thus there is no flotation cost.c. Flotation costs for preferred stock are higher than for debt.d. Firms always pay dividends on their common stock issues because of the ease with which

common shareholders can assume control of the firm. S, S & S

Imputed Costs vs. Explicit Costs64. All of the following are examples of imputed costs except (M)

a. The stated interest paid on a bank loan.b. Assets that are considered obsolete that maintain a net book value.c. Decelerated depreciation. d. Lending funds to a supplier at a lower-than-market rate in exchange for receiving the

supplier’s products at a discount. CMA 0689 1-25

65. The explicit cost of debt financing is the interest expense. The implicit cost(s) of debt financing is (are) the (D)a. Increase in the cost of debt as the debt-to-equity ratio increases.b. Increases in the cost of debt and equity as the debt-to-equity ratio increases.c. Increase in the cost of equity as the debt-to-equity ratio decreases. CMA 1291 1-2d. Decrease in the weighted-average cost of capital as the debt-to-equity ratio increases.

RPCPA, AICPA. CMA & CIA EXAMINATION QUESTIONS Page 12 of 55

Page 13: T04 - Risks & Cost of Capital

MANAGEMENT ADVISORY SERVICES RISKS

Cost of Debt Capital66. Which of the following statements is most correct? (E)

a. Since the money is readily available, the cost of retained earnings is usually a lot cheaper than the cost of debt financing.

b. When calculating the cost of preferred stock, a company needs to adjust for taxes, because preferred stock dividends are tax deductible.

c. When calculating the cost of debt, a company needs to adjust for taxes, because interest payments are tax deductible.

d. Statements a and b are correct.e. Statements b and c are correct. Brigham

67. In computing the cost of capital, the cost of debt capital is determined by (E)a. Annual interest payment divided by the proceeds from debt issuance.b. Interest rate times (1 – the firm’s tax rate)c. Annual interest payment divided by the book value of the debt.d. The capital asset pricing model. Gleim

68. If k is the cost of debt and t is the marginal tax rate, the after-tax cost of debt k, is best represented by the formulaa. ki = k/t c. ki = k(t)b. ki =k/(1 – t) d. ki = k (1 – t) CMA 1288 1-3

69. If Brewer Corporation's bonds are currently yielding 8% in the marketplace, why is the firm's cost of debt lower? (E)A. Market interest rates have increased. B. Additional debt can be issued more cheaply than the original debt. C. There should be no difference; cost of debt is the same as the bonds' market yield. D. Interest is deductible for tax purposes. CMA 0692 1-12

70 The interest rate on the bonds is greater for the second alternative consisting of pure debt than it is for the first alternative consisting of both debt and equity because A. The diversity of the combination alternative creates greater risk for the investor. B. The pure debt alternative would flood the market and be more difficult to sell. C. The pure debt alternative carries the risk of increasing the probability of default. D. The combination alternative carries the risk of increasing dividend payments.

71 If a $1,000 bond sells for $1,125, which of the following statements are correct?I. The market rate of interest is greater than the coupon rate on the bond.II. The coupon rate on the bond is greater than the market rate of interest.III. The coupon rate and the market rate are equal.IV. The bond sells at a premium.

V. The bond sells at a discount.a. I and IV. c. II and IV. b. I and V. d. II and V. CMA 0695 1-6

*. If the return on total assets is 10% and if the return on common stockholders’ equity is 12% then (D)a. The after-tax cost of long-term debt is probably greater than 10%.b. The after-tax cost of long-term debt is 12%.c. Leverage is negative.d. The after-tax cost of long-term debt is probably less than 10%. RPCPA 1095

. Which of the following statements is most correct? (M)a. Suppose a firm is losing money and thus, is not paying taxes, and that this situation is

expected to persist for a few years whether or not the firm uses debt financing. Then the firm’s after-tax cost of debt will equal its before-tax cost of debt.

b. The component cost of preferred stock is expressed as kp(1 - T), because preferred stock dividends are treated as fixed charges, similar to the treatment of debt interest.

c. The reason that a cost is assigned to retained earnings is because these funds are already earning a return in the business; the reason does not involve the opportunity cost principle. Brigham

d. The bond-yield-plus-risk-premium approach to estimating a firm’s cost of common equity involves adding a subjectively determined risk-premium to the market risk-free bond rate.

Marginal Cost of Debt72. The marginal cost of debt for a firm is defined as the interest rate on <List A> debt minus the

<List B>. (M)CIA 0594 IV-48 List A List BA. New Firm's marginal tax rateB. Outstanding Firm's marginal tax rateC. New Interest rate times the firm's marginal tax rateD. Outstanding Interest rate times the firm's marginal tax rate

73. Which of the following statements is most correct? (E)a. If a company’s tax rate increases but the yield to maturity of its noncallable bonds remains

the same, the company’s marginal cost of debt capital used to calculate its weighted average cost of capital will fall.

b. All else equal, an increase in a company’s stock price will increase the marginal cost of retained earnings, ks.

c. All else equal, an increase in a company’s stock price will increase the marginal cost of issuing new common equity, ke.

d. Statements a and b are correct.RPCPA, AICPA. CMA & CIA EXAMINATION QUESTIONS Page 13 of 55

Page 14: T04 - Risks & Cost of Capital

MANAGEMENT ADVISORY SERVICES RISKS

e. Statements b and c are correct. Brigham

Cost of Debt & Cost of Preferred Stock17. The basis for measuring the cost of capital derived from bonds and preferred stock,

respectively, is the (M)A. after-tax rate of interest for bonds and stated annual dividend rate for preferred stock B. pretax rate of interest for bonds and stated annual dividend rate less the expected earnings per share for preferred stock C. pretax rate of interest for bonds and stated annual dividend rate for preferred stock D. after-tax rate of interest for bonds and stated annual dividend rate less the expected earnings per share for preferred stock AICPA adapted

Cost of Debt vs. Cost of Equity Capital74. In general, it is more expensive for a company to finance with equity capital than with debt

capital because (E)A. Long-term bonds have a maturity date and must therefore be repaid in the future. B. Investors are exposed to greater risk with equity capital. C. Equity capital is in greater demand than debt capital. D. Dividends fluctuate to a greater extent than interest rates. CMA 0690 1-15

Cost of Equity CapitalCost of Preferred Stock75. Which of the following statements is most correct? (M)

a. The before-tax cost of preferred stock may be lower than the before-tax cost of debt, even though preferred stock is riskier than debt.

b. If a company’s stock price increases, this increases its cost of common equity.c. If the cost of equity capital is low enough, it may be cheaper to issue common stock than it

is to finance projects with retained earnings.d. Statements a and b are correct. Brigham

Cost of Common Equity. Which of the following factors in the discounted cash flow (DCF) approach to estimating the

cost of common equity is the least difficult to estimate? (E)a. Expected growth rate, g. c. Required return, ks.b. Dividend yield, D1/P0. d. Expected rate of return, . Brigham

76. Assume that nominal interest rates just increased substantially but that the expected future dividends for a company over the long run were not affected. As a result of the increase in nominal interest rates, the company's stock price should A. Increase. C. Stay constant. CIA 0593 IV-49B. Decrease. D. Change, but in no obvious direction.

77. The market value of a firm’s outstanding common shares will be higher, everything else equal, if (M)a. Investors have a lower required return on equity.b. Investors expect lower dividend growth.c. Investors have longer expected holding periods.d. Investors have shorter expected holding periods. CIA 1196 IV-25

Cost of Retained Earnings78. When calculating the cost of capital, the cost assigned to retained earnings should be (E)

A. Zero. B. Lower than the cost of external common equity. C. Equal to the cost of external common equity. D. Higher than the cost of external common equity. CIA 1195 IV-43

79. Which of the following statements is most correct? (M)a. The cost of retained earnings is the rate of return stockholders require on a firm’s common

stock.b. The component cost of preferred stock is expressed as kp(1 - T), because preferred stock

dividends are treated as fixed charges, similar to the treatment of debt interest.c. The bond-yield-plus-risk-premium approach to estimating a firm’s cost of common equity

involves adding a subjectively determined risk-premium to the market risk-free bond rate.d. The higher the firm’s flotation cost for new common stock, the more likely the firm is to use

preferred stock, which has no flotation cost. Brigham

Marginal Cost of Capital80. If a company has a higher dividend-payout ratio, then, if all else if equal, it will have

a. A higher marginal cost of capital.b. A lower marginal cost of capital.c. A higher investment opportunity schedule.d. A lower investment opportunity schedule. CIA 0597 IV-53

81. The firm’s marginal cost of capital (E)a. Should be the same as the firm’s rate of return on equity.b. Is unaffected by the firm’s capital structure. CMA 1291 1-8c. In inversely related to the firm’s required rate of return used in capital budgeting.d. Is a weighted-average of the investors’ required returns on debt and equity.

Dividend Growth Model82. Which of the following criteria theoretically should be used to determine the valuation of

common stock? (E)

RPCPA, AICPA. CMA & CIA EXAMINATION QUESTIONS Page 14 of 55

Page 15: T04 - Risks & Cost of Capital

MANAGEMENT ADVISORY SERVICES RISKS

A. Book value. C. Beta coefficient.B. Dividends. D. Standard deviation of returns. Gleim

83. Which of the following is directly applied in determining the value of a stock when using the dividend growth model? A. The firm's capital structure. B. The firm's cash flows. C. The firm's liquidity. D. The investor's required rate of return on the firm's stock. CIA 1190 IV-53

84. The three elements needed to estimate the cost of equity capital for use in determining a firm's weighted-average cost of capital are (E)A. Current dividends per share, expected growth rate in dividends per share, and current

book value per share of common stock. B. Current earnings per share, expected growth rate in dividends per share, and current

market price per share of common stock. C. Current earnings pers share, expected growth rate in earnings per share, and current

book value per share of common stock. D. Current dividends per share, expected growth rate in dividends per share, and current

market price per share of common stock. CMA 1291 1-3

25. The value of the stock: A. Increases as the dividend growth rate increases B. Increases as the required rate of return decreases C. Increases as the required rate of return increases D. Both A and B B & M

Dividend Growth Rate14. Dividend growth rate for a stable firm can be estimated as:

A. Plow back rate * the return on equity (ROE) B. Plow back rate / the return on equity (ROE) C. Plow back rate +the return on equity (ROE) D. Plow back rate - the return on equity (ROE) B & M

24. The growth rate in dividends can be thought of as a sum of two parts. They are: A. ROE and the Retention Ratio. B. Dividend yield and growth rate in dividends C. ROA and ROE D. Book value per share and EPS B & M

Dividend Growth Model Formula12. The required rate of return on the market capitalization rate is estimated as follows:

A. Dividend yield + expected rate of growth in dividends B. Dividend yield - expected rate of growth in dividends C. Dividend yield / expected rate of growth in dividends D. (Dividend yielD. * (expected rate of growth in dividends) B & M

Capital Asset Pricing Model (CAPM)49. The acronym CAPM stands for:

A. Capital Asset Pricing Model C. Current Arbitrage Pricing Method B & MB. Certainty Asset Pricing Method D. Cumulative Arbitrage Pricing Model

85. The capital asset pricing model assumes (E)a. all investors are price takers.b. all investors have the same holding period.c. investors pay taxes on capital gains.d. both a and b are true.e. a, b and c are all true. Bodie

33. If investors do not know their investment horizons for certain (M)a. the CAPM is no longer valid.b. the CAPM underlying assumptions are not violated.c. the implications of the CAPM are not violated as long as investors’ liquidity needs re not

priced.d. the implications of the CAPM are no longer useful. Bodie

37. The capital asset pricing model (CAPM) states that: A. The expected risk premium on an investment is proportional to its beta B. The expected rate of return on an investment is proportional to its beta C. The expected rate of return on an investment depends on the risk-free rate and the

market rate of return B & MD. The expected rate of return on an investment is dependent on the risk-free rate

56. The drawback of the CAPM is that it: A. Ignores the return on the market portfolio B. Required a single measure of systematic risk C. Ignores risk-free return D. Utilizes too many factors B & M

Security Market Line46. The security market line (SML) shows the relationship between.

RPCPA, AICPA. CMA & CIA EXAMINATION QUESTIONS Page 15 of 55

Page 16: T04 - Risks & Cost of Capital

MANAGEMENT ADVISORY SERVICES RISKS

A. Expected return and standard deviation B. Expected return and beta C. Standard deviation and risk D. Variance and beta

38. The security market line (SML) is the graph of: A. Expected rate on investment (Y-axis) vs. variance of return B. Expected return on investment vs. standard deviation of return C. Expected rate of return on investment vs. beta D. A and B B & M

86 The Security Market Line (SML) is (M)a. the line that describes the expected return-beta relationship for well-diversified portfolios only.b. also called the Capital Allocation Line.c. the line that is tangent to the efficient frontier of all risky assets.d. the line that represents the expected return-beta relationship.e. the line that represents the relationship between an individual security’s return and the

market’s return. Bodie

87. The security market line (SML) (M)a. can be portrayed graphically as the expected return-beta relationship.b. can be portrayed graphically as the expected return-standard deviation of market returns

relationship.c. provides a benchmark for evaluation of investment performance.d. a and c.e. b and c. Bodie

88 Which statement is not true regarding the Capital Market Line (CML)? (M)a. The CML is the line from the risk-free rate through the market portfolio.b. The CML is the best attainable capital allocation line.c. The CML is also called the security market line.d. The CML always has a positive slope. Bodie

89. In equilibrium, the marginal price of risk for a risky security must be (M)a. equal to the marginal price of risk for the market portfolio.b. greater than the marginal price of risk for the market portfolio.c. less than the marginal price of risk for the market portfolio.d. adjusted by its degree of nonsystematic risk. Bodie

90. An underpriced security will plot (E)a. on the Security Market Line.

b. below the Security Market Line.c. above the Security Market Line. Bodied. either above or below the Security Market Line depending on its covariance with the arket.e. either above or below the Security Market Line depending on its standard deviation.

47. If a stock is overpriced it would plot: A. Above the security market line C. Below the security market lineB. On the security market line D. On the Y-axis B & M

Variables91. An analysis of a company’s planned equity financing using the capital asset pricing model (or

security market line) would incorporate only thea. Expected market earnings, the current U.S. Treasury bond yield, and the beta coefficient.b. Expected market earnings and the price-earnings ratio.c. Current U.S. Treasury bond yield, the price-earnings ratio, and the beta coefficient.d. Current U.S. Treasury bond yield and the dividend payout ratio. CMA 1291 1-16

Formula92 According to the Capital Asset Pricing Model (CAPM), the expected rate of return on any security

is equal to (M)a. Rf + [E(RM)]. c. Rf + [E(RM - Rf].b. Rf + [E(RM) - Rf]. d. E(RM) + Rf. Bodie

Market Portfolio93 Which statement is not true regarding the market portfolio? (M)

a. It includes all assets of the universe.b. It lies on the efficient frontier. Bodiec. All securities in the market portfolio are held in proportion to their market values.d. It is the tangency point between the capital market line and the indifference curve.

Alpha Coefficient94 According to the Capital Asset Pricing Model (CAPM), fairly priced securities (M)

a. have positive betas. c. have negative betas.b. have zero alphas. d. have positive alphas. Bodie

95. According to the Capital Asset Pricing Model (CAPM), (M)a. a security with a positive alpha is considered overpriced.b. a security with a zero alpha is considered to be a good buy.c. a security with a negative alpha is considered to be a good buy.d. a security with a positive alpha is consider to be underpriced. Bodie

RPCPA, AICPA. CMA & CIA EXAMINATION QUESTIONS Page 16 of 55

Page 17: T04 - Risks & Cost of Capital

MANAGEMENT ADVISORY SERVICES RISKS

Beta Coefficient96. A measure that describes the risk of an investment project relative to other investments in

general is the (E)A. Coefficient of variation. C. Standard deviation.B. Beta coefficient. D. Expected return. CIA 1187 IV-66

97. What is the formula for the beta coefficient of a security? A. Covariance of the returns on the market and on the security ÷ Variance of the return on

the market. B. Covariance of the returns on the market and on the security x Variance of the return on

the market. C. Variance of the return on the market ÷ Variance of the return on the security. D. Variance of the return on the market x Variance of the return on the security ÷ Covariance

of the returns on the market and on the security. Gleim

98. According to the capital asset pricing model (CAPM), the relevant risk of a security is its A. Company-specific risk. C. Systematic risk.B. Diversifiable risk. D. Total risk. CIA 1194 IV-53

39. Beta measures: A. The ability to diversify risk B. The change in the rate of return of an investment for a given change in the market rate of

return C. The actual return on an asset D. A and C B & M

40. Beta measure indicates: A. The ability to diversify risk B. The change in the rate of return on an investment for a given change in the market return C. The actual return on an asset D. A and C B & M

99 The market risk, beta, of a security is equal to (M)a. the covariance between the security’s return and the market return divided by the variance of

the market's returns.b. the covariance between the security and market returns divided by the standard deviation of

the market's returns.c. the variance of the security's returns divided by the covariance between the security and

market returns. Bodied. the variance of the security's returns divided by the variance of the market's returns.

100 The market portfolio has a beta of (E)a. 0. c. -1.b. 1. d. 0.5. Bodie

. If the firm is being operated so as to maximize shareholder wealth, and if our basic assumptions concerning the relationship between risk and return are true, then which of the following should be true? (M)a. If the beta of the asset is larger than the firm’s beta, then the required return on the asset

is less than the required return on the firm.b. If the beta of the asset is smaller than the firm’s beta, then the required return on the asset

is greater than the required return on the firm.c. If the beta of the asset is greater than the firm’s beta prior to the addition of that asset,

then the firm’s beta after the purchase of the asset will be smaller than the original firm’s beta.

d. If the beta of an asset is larger than the firm’s beta prior to the addition of that asset, then the required return on the firm will be greater after the purchase of that asset than prior to its purchase. Brigham

. Which of the following statements is most correct? (M)a. Beta measures market risk, but if a firm’s stockholders are not well diversified, beta may

not accurately measure stand-alone risk.b. If the calculated beta underestimates the firm’s true investment risk, then the CAPM

method will overestimate ks.c. The discounted cash flow method of estimating the cost of equity can’t be used unless the

growth component, g, is constant during the analysis period. Brighamd. An advantage shared by both the DCF and CAPM methods of estimating the cost of

equity capital, is that they yield precise estimates and require little or no judgement.

. Which of the following statements is correct? (M)a. The cost of capital used to evaluate a project should be the cost of the specific type of

financing used to fund that project.b. The cost of debt used to calculate the weighted average cost of capital is based on an

average of the cost of debt already issued by the firm and the cost of new debt.c. One problem with the CAPM approach to estimating the cost of equity capital is that if a

firm’s stockholders are, in fact, not well diversified, beta may be a poor measure of the firm’s true investment risk.

d. The bond-yield-plus-risk-premium approach is the most sophisticated and objective method of estimating a firm’s cost of equity capital.

e. The cost of equity capital is generally easier to measure than the cost of debt, which varies daily with interest rates, or the cost of preferred stock since preferred stock is issued infrequently. Brigham

RPCPA, AICPA. CMA & CIA EXAMINATION QUESTIONS Page 17 of 55

Page 18: T04 - Risks & Cost of Capital

MANAGEMENT ADVISORY SERVICES RISKS

101. In the context of the Capital Asset Pricing Model (CAPM) the relevant measure of risk is (E)a. unique risk. c. standard deviation of returns.b. beta. d. variance of returns. Bodie

45. A stock with a beta of 1.2 would be expected to: A. Increase 20% faster than the market in up markets B. Increase 20% faster than the market in down markets C. Increase 120% faster than the market in up markets D. Increase 120% faster than the market in down markets B & M

Risk-free Rate102. What is the expected return of a zero-beta security? (M)

a. The market rate of return. c. A negative rate of return.b. Zero rate of return. d. The risk-free rate. Bodie

Risk Premium103. According to the CAPM, the risk premium an investor expects to receive on any stock or portfolio

increases: (E)a. directly with alpha. d. inversely with beta.b. inversely with alpha. e. in proportion to its standard deviation.c. directly with beta. Bodie

104. The risk premium on the market portfolio will be proportional to (M)a. the average degree of risk aversion of the investor population.b. the risk of the market portfolio as measured by its variance.c. the risk of the market portfolio as measured by its beta.d. both a and b are true.e. both a and c are true. Bodie

36. The market risk premium is: A. The difference between the rate of return on an asset and the risk-free rate B. The difference between the rate of return on the market portfolio and the risk-free rate C. The risk-free rate D. The market rate of return B & M

Pure Play Method. Which of the following methods involves calculating an average beta for firms in a similar

business and then applying that beta to determine a project’s beta? (M)a. Risk premium method. c. Accounting beta method.b. Pure play method. d. CAPM method. Brigham

105. Interstate Transport has a target capital structure of 50 percent debt and 50 percent common equity. The firm is considering a new independent project that has an expected return of 13 percent and is not related to transportation. However, a pure play proxy firm has been identified that is exclusively engaged in the new line of business. The proxy firm has a beta of 1.38. Both firms have a marginal tax rate of 40 percent, and Interstate’s before-tax cost of debt is 12 percent. The risk-free rate is 10 percent and the market risk premium is 5 percent. The firm should(M)a. Reject the project; its expected return is less than the firm’s required rate of return on the

project of 16.9 percent.b. Accept the project; its expected return is greater than the firm’s required rate of return on

the project of 12.05 percent.c. Reject the project; its expected return is only 13 percent.d. Accept the project; its expected return exceeds the risk-free rate and the before-tax cost

of debt.e. Be indifferent between accepting or rejecting; the firm’s required rate of return on the

project equals its expected return. Brigham

Required Rate of Return106. If the return on the market portfolio is 10% and the risk-free rate is 5%, what is the effect on a

company's required rate of return on its stock of an increase in the beta coefficient from 1.2 to 1.5? A. 3% increase C. No changeB. 1.5% increase D. 1.5% decrease Gleim

Expected Rate of Return vs. Required Rate of Return107. Security X has an expected rate of return of 0.11 and a beta of 1.5. The risk-free rate is 0.05 and

the market expected rate of return is 0.09. According to the Capital Asset Pricing Model, this security is (M)a. underpriced.b. overpriced.c. fairly priced.d. cannot be determined from data provided. Bodie

108. The risk-free rate is 7 percent. The expected market rate of return is 15 percent. If you expect stock X with a beta of 1.3 to offer a rate of return of 12 percent, you should (M)a. buy stock X because it is overpriced.b. sell short stock X because it is overpriced.c. sell stock short X because it is underpriced.d. buy stock X because it is underpriced. Bodie

RPCPA, AICPA. CMA & CIA EXAMINATION QUESTIONS Page 18 of 55

Page 19: T04 - Risks & Cost of Capital

MANAGEMENT ADVISORY SERVICES RISKS

109. Given the following two stocks A and BExpected rate of return Beta

A 0.12 1.2B 0.14 1.8

If the expected market rate of return is 0.09 and the risk-free rate is 0.05, which security would be considered the better buy and why? (M)a. A because it offers an expected excess return of 1.2%.b. B because it offers an expected excess return of 1.8%.c. A because it offers an expected excess return of 2.2%.d. B because it offers an expected return of 14%.e. B because it has a higher beta. Bodie

Weighted-Average Cost of Capital (WACC)1. The cost of capital is defined as (E)

a. the simple average of the interest rates of all debt outstanding.b. the simple average of the cost of debt and equity.c. the weighted average of the interest rates of all debt outstanding.d. the weighted average of the cost of debt and equity. S, S & S

. Which of the following is not considered a capital component for the purpose of calculating the weighted average cost of capital as it applies to capital budgeting? (E)a. Long-term debt. c. Short-term debt.b. Common stock. d. Preferred stock. Brigham

28. The combined weighted average interest rate that a firm incurs on its long-term debt, preferred stock, and common stock is thea. cost of capital. c. cutoff rate.b. discount rate. d. internal rate of return. Barfield

108.The weighted average cost of capital represents thea. cost of bonds, preferred stock, and common stock divided by the three sources.b. equivalent units of capital used by the organization.c. overall cost of capital from all organization financing sources.d. overall cost of dividends plus interest paid by the organization. Barfield

29. The weighted average cost of capital that is used to evaluate a specific project should be based on thea. mix of capital components that was used to finance a project from last year.b. overall capital structure of the corporation.c. cost of capital for other corporations with similar investments.d. mix of capital components for all capital acquired in the most recent fiscal year. Barfield

110. Which of the following statements is correct? (M)a. Because we often need to make comparisons among firms that are in different income tax

brackets, it is best to calculate the WACC on a before-tax basis.b. If a firm has been suffering accounting losses and is expected to continue suffering such

losses, and therefore its tax rate is zero, it is possible that its after-tax component cost of preferred stock as used to calculate the WACC will be less than its after-tax component cost of debt.

c. Normally, the cost of external equity raised by issuing new common stock is above the cost of retained earnings. Moreover, the higher the growth rate is relative to the dividend yield, the more the cost of external equity will exceed the cost of retained earnings.

d. The lower a company’s tax rate, the greater the advantage of using debt in terms of lowering its WACC. Brigham

111. When calculating a firm's cost of capital, all of the following are true except that (E)A. The cost of capital of a firm is the weighted average cost of its various financing

components. B. The calculation of the cost of capital should focus on the historical costs of alternative

forms of financing rather than market or current costs. C. All costs should be expressed as after-tax costs. D. The time value of money should be incorporated into the calculations. CMA 1288 1-2

112. A company has made the decision to finance next year's capital projects through debt rather than additional equity. The benchmark cost of capital for these projects should be (M)A. The before-tax cost of new-debt financing. B. The after-tax cost of new-debt financing. C. The cost of equity financing. D. The weighted-average cost of capital. CIA 0597 IV-42

113. Which of the following is true regarding the calculation of a firm's cost of capital? (E)A. The cost of capital of a firm is the weighted-average cost of its various financing

components. B. All costs should be expressed as pre-tax costs. C. The time value of money should be excluded from the calculations. D. The cost of capital is the cost of equity. CMA 1288 1-2

18. The weighted-average cost of capital approach to decision making is not directly affected by the: (E)A. proposed mix of debt, equity, and existing funds used to implement the projectB. value of the common stockC. cost of debt outstanding

RPCPA, AICPA. CMA & CIA EXAMINATION QUESTIONS Page 19 of 55

Page 20: T04 - Risks & Cost of Capital

MANAGEMENT ADVISORY SERVICES RISKS

D. current budget for expansion AICPA adapted

30. Debt in the capital structure could be treated as if it were common equity in computing the weighted average cost of capital if the debt werea. callable. c. cumulative.b. participating. d. convertible. Barfield

. For a typical firm with a given capital structure, which of the following is correct? (Note: All rates are after taxes.) (E)a. kd > ke > ks > WACC. c. WACC > ke > ks > kd.b. ks > ke > kd > WACC. d. ke > ks > WACC > kd. Brigham

33. Which of the following is true? A. rD < rA < rE C. rE < rA < rDB. rE < rD < rA D. None of the above are true B & M

31. Generally, which of the following is true? A. rD > rA > rE C. rE > rA > rDB. rE > rD > rA D. None of the above are true B & M

114. Which of the following statements is correct? (M)a. The WACC should include only after-tax component costs. Therefore, the required rates

of return (or “market rates”) on debt, preferred, and common equity (kd, kp, and ks) must be adjusted to an after-tax basis before they are used in the WACC equation. Brigham

b. The cost of retained earnings is generally higher than the cost of new common stock.c. Preferred stock is riskier to investors than is debt. Therefore, if someone told you that the

market rates showed kd > kp for a given company, that person must have made a mistake.d. If a company with a debt ratio of 50 percent were suddenly exempted from all future

income taxes, then, all other things held constant, this would cause its WACC to increase.

115. Which of the following statements is most correct? (M)a. The weighted average cost of capital for a given capital budget level is a weighted

average of the marginal cost of each relevant capital component that makes up the firm’s target capital structure.

b. The weighted average cost of capital is calculated on a before-tax basis.c. An increase in the risk-free rate is likely to increase the marginal costs of both debt and

equity financing.d. Statements a and c are correct. Brigham

116. A company has a capital structure that consists of 50 percent debt and 50 percent equity. Which of the following statements is most correct? (E)a. The cost of equity financing is greater than or equal to the cost of debt financing.b. The WACC exceeds the cost of equity financing.c. The WACC is calculated on a before-tax basis.d. The WACC represents the cost of capital based on historical averages. In that sense, it

does not represent the marginal cost of capital.e. The cost of retained earnings exceeds the cost of issuing new common stock. Brigham

117. Which of the following statements is most correct? (E)a. The WACC is a measure of the before-tax cost of capital.b. Typically the after-tax cost of debt financing exceeds the after-tax cost of equity financing.c. The WACC measures the marginal after-tax cost of capital.d. Statements a and b are correct.e. Statements b and c are correct. Brigham

118. Campbell Co. is trying to estimate its weighted average cost of capital (WACC). Which of the following statements is most correct? (E)a. The after-tax cost of debt is generally cheaper than the after-tax cost of preferred stock.b. Since retained earnings are readily available, the cost of retained earnings is generally

lower than the cost of debt.c. If the company’s beta increases, this will increase the cost of equity financing, even if the

company is able to rely on only retained earnings for its equity financing.d. Statements a and b are correct.e. Statements a and c are correct. Brigham

119. Which of the following statements is most correct? (E)a. The WACC represents the after-tax cost of capital.b. The WACC represents the marginal cost of capital.c. The cost of retained earnings is generally more expensive than the cost of issuing new

common stock, because it includes an opportunity cost.d. Statements a and b are correct.e. All of the statements above are correct. Brigham

120. Which of the following statements about the cost of capital is incorrect? (E)a. A company’s target capital structure affects its weighted average cost of capital.b. Weighted average cost of capital calculations should be based on the after-tax costs of all

the individual capital components.c. If a company’s tax rate increases, then, all else equal, its weighted average cost of capital

will increase.d. Flotation costs can increase the weighted average cost of capital.

RPCPA, AICPA. CMA & CIA EXAMINATION QUESTIONS Page 20 of 55

Page 21: T04 - Risks & Cost of Capital

MANAGEMENT ADVISORY SERVICES RISKS

e. An increase in the risk-free rate is likely to increase the marginal costs of both debt and equity financing. Brigham

. A company estimates that an average-risk project has a WACC of 10 percent, a below-average risk project has a WACC of 8 percent, and an above-average risk project has a WACC of 12 percent. Which of the following independent projects should the company accept? (E)a. Project A has average risk and a return of 9 percent.b. Project B has below-average risk and a return of 8.5 percent.c. Project C has above-average risk and a return of 11 percent.d. All of the projects above should be accepted.e. None of the projects above should be accepted. Brigham

22. Which of the following regarding the weighted-average cost of capital is true?a. The tax effect of preferred stock dividends should be included in the calculation of

weighted-average cost of capital.b. The tax effect of common stock dividends should be included in the calculation of

weighted-average cost of capital.c. The tax effect of debt should be included in the calculation of the weighted-average cost

of capital.d. Taxes do not affect the weighted-average cost of capital. S, S & S

23. Which of the following is true regarding the weighted-average cost of capital?a. A company may have two weighted-average costs of capital if the firm's capital structure

is so large that new common stock must be sold.b. The book value of the components of capital should always be used to calculate the

weighted-average cost of capital.c. The cost of common equity is lower than the cost of retained earnings.d. The cost of preferred stock is adjusted for the tax deduction associated with preferred

dividends. S, S & S

WACC in a Tax-Free Environment21. The cost of capital for a firm, rWACC, in a tax free environment is:

A. Equal to the expected EBIT divided by market value of the unlevered firm B. Equal to rA, the rate of return for that business risk class C. Equal to the overall rate of return required on the levered firm D. All of the above B & M

Capital Asset Pricing Model121. Wyden Brothers has no retained earnings. The company uses the CAPM to calculate the cost

of equity capital. The company’s capital structure consists of common stock, preferred stock, and debt. Which of the following events will reduce the company’s WACC? (M)a. A reduction in the market risk premium.b. An increase in the flotation costs associated with issuing new common stock.c. An increase in the company’s beta.d. An increase in expected inflation.e. An increase in the flotation costs associated with issuing preferred stock. Brigham

Required Rate of Return122. Louisiana Enterprises, an all-equity firm, is considering a new capital investment. Analysis has

indicated that the proposed investment has a beta of 0.5 and will generate an expected return of 7 percent. The firm currently has a required return of 10.75 percent and a beta of 1.25. The investment, if undertaken, will double the firm’s total assets. If kRF is 7 percent and the market return is 10 percent, should the firm undertake the investment? (Choose the best answer.) (D)a. Yes; the expected return of the asset (7%) exceeds the required return (6.5%).b. Yes; the beta of the asset will reduce the risk of the firm.c. No; the expected return of the asset (7%) is less than the required return (8.5%).d. No; the risk of the asset (beta) will increase the firm’s beta.e. No; the expected return of the asset is less than the firm’s required return, which is

10.75%. Brigham

123. Interstate Transport has a target capital structure of 50 percent debt and 50 percent common equity. The firm is considering a new independent project that has a return of 13 percent and is not related to transportation. However, a pure play proxy firm has been identified that is exclusively engaged in the new line of business. The proxy firm has a beta of 1.38. Both firms have a marginal tax rate of 40 percent, and Interstate’s before-tax cost of debt is 12 percent. The risk-free rate is 10 percent and the market risk premium is 5 percent. The firm should(M)a. Reject the project; its return is less than the firm’s required rate of return on the project of

16.9 percent.b. Accept the project; its return is greater than the firm’s required rate of return on the project

of 12.05 percent.c. Reject the project; its return is only 13 percent.d. Accept the project; its return exceeds the risk-free rate and the before-tax cost of debt.

RPCPA, AICPA. CMA & CIA EXAMINATION QUESTIONS Page 21 of 55

Page 22: T04 - Risks & Cost of Capital

MANAGEMENT ADVISORY SERVICES RISKS

e. Be indifferent between accepting or rejecting; the firm’s required rate of return on the project equals its expected return. Brigham

Optimal Project Selection124. Jackson Corporation is evaluating the following four independent, investment

opportunities:Project Cost Expected Rate of Return

A $300,000 14%B 150,000 10C 200,000 13D 400,000 11

Jackson’s target capital structure is 60 percent debt and 40 percent equity. The yield to maturity on the company’s debt is 10 percent. Jackson will incur flotation costs for a new equity issuance of 12 percent. The growth rate is a constant 6 percent. The stock price is currently $35 per share for each of the 10,000 shares outstanding. Assume that the firm has no retained earnings. If the company’s tax rate is 30 percent, then which of the projects will be accepted? (M)a. Project A. c. Projects A, C, and D. Brighamb. Projects A and C. d. All of the investment projects will be taken.

Sensitivity AnalysisRisk, Required Return and project betas. If the firm is being operated so as to maximize shareholder wealth, and if our basic

assumptions concerning the relationship between risk and return are true, then which of the following should be true? (M)a. If the beta of the asset is larger than the firm’s beta, then the required return on the asset

is less than the required return on the firm.b. If the beta of the asset is smaller than the firm’s beta, then the required return on the asset

is greater than the required return on the firm.c. If the beta of the asset is greater than the corporate beta prior to the addition of that asset,

then the corporate beta after the purchase of the asset will be smaller than the original corporate beta.

d. If the beta of an asset is larger than the corporate beta prior to the addition of that asset, then the required return on the firm will be greater after the purchase of that asset than prior to its purchase. Brigham

Beta and project risk. Which of the following statements is correct? (D)

a. A relatively risky future cash outflow should be evaluated using a relatively low discount rate.

b. If a firm’s managers want to maximize the value of the stock, they should concentrate exclusively on projects’ market, or beta, risk.

c. If a firm evaluates all projects using the same cost of capital, then the riskiness of the firm as measured by its beta will probably decline over time.

d. If a firm has a beta that is less than 1.0, say 0.9, this would suggest that its assets’ returns are negatively correlated with the returns of most other firms’ assets. Brigham

Comprehensive. Which of the following statements is correct? (M)

a. Although some methods of estimating the cost of equity capital encounter severe difficulties, the CAPM is a simple and reliable model that provides great accuracy and consistency in estimating the cost of equity capital.

b. The DCF model is preferred over other models to estimate the cost of equity because of the ease with which a firm’s growth rate is obtained.

c. The bond-yield-plus-risk-premium approach to estimating the cost of equity is not always accurate but its advantages are that it is a standardized and objective model.

d. Depreciation-generated funds are an additional source of capital and, in fact, represent the largest single source of funds for some firms. Brigham

LEVERAGELimits of leverage21. Which of the following are practical difficulties associated with capital structure and degree of

leverage analyses? (M)a. It is nearly impossible to determine exactly how P/E ratios or equity capitalization rates (k s

values) are affected by different degrees of financial leverage.b. Managers’ attitudes toward risk differ and some managers may set a target capital

structure other than the one that would maximize stock price.c. Managers often have a responsibility to provide continuous service; they must preserve

the long-run viability of the enterprise. Thus, the goal of employing leverage to maximize short-run stock price and minimize capital cost may conflict with long-run viability.

d. All of the statements above are correct.e. None of the statements above represents a serious impediment to the practical

application of leverage analysis in capital structure determination. Brigham

Operating Leverage125. Which class of leverage causes earnings before interest and taxes to be more sensitive to

changes in sales? (M)A. Credit. C. Operating.B. Financial. D. Intrinsic. CIA 0593 IV-57

RPCPA, AICPA. CMA & CIA EXAMINATION QUESTIONS Page 22 of 55

Page 23: T04 - Risks & Cost of Capital

MANAGEMENT ADVISORY SERVICES RISKS

126. The percentage change in earnings before interest and taxes associated with the percentage change in sales volume represents the degree of (E)A. Operating leverage. C. Breakeven leverage.B. Financial leverage. D. Combined leverage. CIA 1189 IV-54

127. The degree of operating leverage (DOL) is A. Constant at all levels of sales. B. A measure of the change in earnings available to common stockholders associated with a

given change in operating earnings. C. A measure of the change in operating income resulting from a given change in sales. D. Lower if the degree of total leverage is higher, other things held constant. CIA 0594 IV-52

128. Companies experience changes in interest expenses, variable cost per unit, quantity of units sold, and fixed costs. Their degree of operating leverage is not affected by the change in A. Interest expenses. C. Quantity of units sold.B. Variable cost per unit. D. Fixed costs. CIA 1193 IV-53

129. A firm with a higher degree of operating leverage when compared to the industry average implies that the A. Firm has higher variable costs. B. Firm's profits are more sensitive to changes in sales volume. C. Firm is more profitable. D. Firm is less risky. CMA 0695 1-1

Financial Leverage*. Securing the funds for investment at a fixed rate of return to fund suppliers, to enhance the

well-being of the common stockholders is known asa. Financial leverage. c. Prudent borrowing.b. Fund management. d. Financial arbitrage. RPCPA 1097

*. It refers to the practice of financing assets with borrowed capital. Its extensive use may impact on the return on common stockholders’ equity to be above or below the rate of return on total assets.a. Discounting. c. Leverage.b. Mortgage. d. Arbitrage. RPCPA 0597

*. “Trading on equity” pertains to the practice of (E) RPCPA 1077, 0588a. Trading in the stock of other companies c. Using funds provided by creditorsb. Retiring bonds before maturity d. Purchasing and selling treasury stocks

*. This accounting terminology has reference to long-term debt and means that you borrow somebody’s money at an interest rate which is lower than the rate which you can earn on that money.a. Pooling of interest. c. Kiting.b. Leverage or trading on equity. d. None of the above. RPCPA 1084

RPCPA, AICPA. CMA & CIA EXAMINATION QUESTIONS Page 23 of 55

Page 24: T04 - Risks & Cost of Capital

MANAGEMENT ADVISORY SERVICES RISKS

*. The use of borrowed capital by business firms is referred to as leverage or trading on equity. This leverage is likely to be a sound financial strategy for stockholders of companies havinga. Cyclical high and low amounts of reported earnings.b. Steady amounts of reported earnings.c. Volatile fluctuation in reported earnings over short periods of time.d. Steadily declining amounts of reported earnings. RPCPA 1079

130. The purchase of treasury stock with a firm's surplus cash CMA 1291 1-5A. Increases a firm's assets. C. Increases a firm's interest coverage ratio.B. Increases a firm's financial leverage. D. Dilutes a firm's earnings per share.

131. When a company increases its degree of financial leverage (DFL)a. The equity beta of the company falls.b. The systematic risk of the company falls.c. The systematic risk of the company rises.d. The standard deviation of returns on the equity of the company rises. CIA 0597 IV-50

132. Sylvan Corporation has the following capital structure.Debenture bonds $10,000,000Preferred equity 1,000,000Common equity 39,000,000

The financial leverage of Sylvan Corporation would increase as a result of (M)A. Issuing common stock and using the proceeds to retire preferred stock. B. Maintaining the same dollar level of cash dividends as the prior year, even though

earnings have increased by 7%. C. Financing its future investments with a higher percentage of bonds. D. Financing its future investments with a higher percentage of equity funds. CMA 0690 1-9

133. Everything else being equal, a <List A> highly leveraged firm will have <List B> earnings per share. (D)CIA 0595 IV-51 A. B. C. D.List A More More Less LessList B Lower Less volatile Less volatile Higher

134. A company is considering the early retirement of its 10%, 10-year bonds payable. Before retiring the bonds, the company's capital structure was

Current liabilities $125,000Long-term liabilities:Notes payable (due in 5 years) 200,000Bonds payable 300,000

Premium on bonds payable 25,000Owner's equity:Common shares ($5 par value) 150,000Share premium in excess of par 50,000Retained earnings 450,000

If the bonds can be retired at 103.5%, the CIA 1193 IV-48A. Debt-equity ratio will increase. C. Asset turnover ratio will decrease.B. Financial leverage will decrease. D. Return on owner's equity will decrease.

Financial leverage and EPS135. Volga Publishing is considering a proposed increase in its debt ratio, which will also increase

the company’s interest expense. The plan would involve the company issuing new bonds and using the proceeds to buy back shares of its common stock. The company’s CFO expects that the plan will not change the company’s total assets or operating income. However, the company’s CFO does estimate that it will increase the company’s earnings per share (EPS). Assuming the CFO’s estimates are correct, which of the following statements is most correct? (E)a. Since the proposed plan increases Volga’s financial risk, the company’s stock price still

might fall even though its EPS is expected to increase.b. If the plan reduces the company’s WACC, the company’s stock price is also likely to

decline.c. Since the plan is expected to increase EPS, this implies that net income is also expected

to increase.d. Statements a and b are correct.e. Statements a and c are correct. Brigham

136. Which of the following statements is most correct? (E)a. Increasing financial leverage is one way to increase a firm’s basic earning power (BEP).b. Firms with lower fixed costs tend to have greater operating leverage.c. The debt ratio that maximizes EPS generally exceeds the debt ratio which maximizes

share price.d. Statements a and b are correct.e. Statements a and c are correct. Brigham

Financial leverage and ratios137. Company A and Company B have the same tax rate, the same total assets, and the same

basic earning power. Both companies have a basic earning power that exceeds their before-tax costs of debt, kd. However, Company A has a higher debt ratio and higher interest expense than Company B. Which of the following statements is most correct? (E)a. Company A has a lower net income than B.b. Company A has a lower ROA than B.

RPCPA, AICPA. CMA & CIA EXAMINATION QUESTIONS Page 24 of 55

Page 25: T04 - Risks & Cost of Capital

MANAGEMENT ADVISORY SERVICES RISKS

c. Company A has a lower ROE than B.d. Statements a and b are correct. Brigham

138 . Firm U and Firm L each have the same total assets. Both firms also have a basic earning power of 20 percent. Firm U is 100 percent equity financed, while Firm L is financed with 50 percent debt and 50 percent equity. Firm L’s debt has a before-tax cost of 8 percent. Both firms have positive net income. Which of the following statements is most correct? (E)a. The two companies have the same times interest earned (TIE) ratio.b. Firm L has a lower ROA.c. Firm L has a lower ROE.d. Statements a and b are correct.e. Statements b and c are correct. Brigham

Financial leverage and ratios139. Company A and Company B have the same total assets, operating income (EBIT), tax rate,

and business risk. Company A, however, has a much higher debt ratio than Company B. Company A’s basic earning power (BEP) exceeds its cost of debt financing (kd). Which of the following statements is most correct? (M)a. Company A has a higher return on assets (ROA) than Company B.b. Company A has a higher times interest earned (TIE) ratio than Company B.c. Company A has a higher return on equity (ROE) than Company B, and its risk, as

measured by the standard deviation of ROE, is also higher than Company B’s.d. Statements b and c are correct. Brigham

Operating and Financial Leverage20. Which of the following statements is most correct? (M)

a. Firms whose sales are very sensitive to changes in the business cycle are more likely to rely on debt financing.

b. Firms with large tax loss carry forwards are more likely to rely on debt financing.c. Firms with a high operating leverage are more likely to rely on debt financing.d. Statements a and c are correct.e. None of the statements above is correct. Brigham

Comprehensive140. Which of the changes in leverage would apply to a company that substantially

increases its investment in fixed assets as a proportion of total assets and replaces some of its long-term debt with equity? (M)CIA 0590 IV-56 A. B. C. D.Financial Leverage Increase Decrease Increase DecreaseOperating Leverage Decrease Increase Increase Decrease

INSURANCE141. Insurance may best be defined as

A. A system for transferring risk through risk avoidance or loss control. B. Any contract that conveys an insurable interest. C. A form of pure risk called gambling. GleimD. A means of combining many loss exposures so that losses are shared by all participants.

142. Which of the following is the best functional definition of insurance? A. A legal contract by which the insurer, in return for consideration, agrees to pay another

person if a stated loss or injury occurs. B. A legal contract by which an insurance company, in return for premiums, agrees to pay

the policyholder if a certain event occurs. C. A written promise by the insurer to pay the beneficiary if loss occurs from the occurrence

of a contingent event. D. A writing issued by an insurance company, for a consideration, that promises to indemnify

a beneficiary for a loss from an existing risk or one which arises later. Gleim

143. In which way does the formation of an insurance contract differ from any other contract? A. The requirement that the insured must have an insurable interest. B. The insurance contract is not valid unless written. C. Consideration is not needed for the formation of an insurance contract. D. In insurance, only the insured can commit a breach. Gleim

144. Life insurance A. Is a contract of indemnity. C. Covers only the mortality risk.B. Usually has short-term policies. D. Generally has no cash value. Gleim

145. Life insurance is offered in several forms. The kind that offers no investment feature is A. Whole life. C. Straight life.B. Endowment. D. Term. Gleim

146. Jon Berstock is an employee of PR, Inc. During his employment, the corporation's earnings have doubled, largely because of Jon's ability to attract new accounts. PR therefore insured his life for a substantial sum. If Jon dies, will PR be able to collect the insurance proceeds? A. Yes, because a corporation has an insurable interest in all its employees since it can act

only through agents. B. Yes, because PR has a pecuniary interest in Jon's continued life. C. No, because PR will continue to exist after Jon's death. D. No, because Jon was not a stockholder or officer of PR, Inc. Gleim

RPCPA, AICPA. CMA & CIA EXAMINATION QUESTIONS Page 25 of 55

Page 26: T04 - Risks & Cost of Capital

MANAGEMENT ADVISORY SERVICES RISKS

147. Which of the following is a characteristic of fire insurance? A. It is more standardized than life insurance. B. It is written for a relatively short period but usually includes an incontestability clause. C. A policy must be valued and contain a pro rata clause. D. The insurable interest must be an ownership interest in the property itself. Gleim

148. The typical life insurance policy contains A. No exclusion for death during military service. B. A clause allowing coverage for death during noncommercial flight. C. A prohibition on reinstatement. D. A provision for a grace period for premium payment. Gleim

149. A fire insurance policy ordinarily indemnifies for losses arising from A. Friendly, but not hostile, fires. B. Hostile, but not friendly, fires. C. Both hostile and friendly fires. D. Smoke produced by friendly or hostile fires. Gleim

150. Which of the following wrongful acts prevents recovery under a policy of fire insurance? A. Arson by the insured's employees or agents. B. Arson by third persons unrelated to the insured. C. An act by the insured intended to cause the damage. D. Gross negligence but not amounting to recklessness and willful misconduct. Gleim

151. Which of the following is not a type of insurance policy that provides liability coverage? A. Malpractice insurance. C. Automobile insurance.B. Homeowners insurance. D. Fire insurance. Gleim

152. The purpose of a co-insurance clause is to A. Encourage policyholders to bear a proportionate part of any loss. B. Encourage insurers to pay the face amount of the policy in the event of a partial loss on

the part of the insured. C. Encourage policyholders to insure commercial property for an amount that is near to its

full replacement cost. D. Encourage policyholders to insure commercial property for an amount that is significantly

less than its full replacement cost. Gleim

153. Jewelry, Inc. took out an insurance policy with Insurance Company which covered the stock of jewelry. Insurance agreed to indemnify for losses due to theft of the jewels displayed. The application contained the following provision: "It is hereby warranted that the maximum value of the jewelry displayed shall not exceed $10,000." The insurance policy's coverage was for $8,000. Subsequently, thieves smashed the store window and stole $4,000 worth of jewels when the total value of the display was $12,000. Which of the following is correct? A. Jewelry, Inc. will recover nothing. B. Jewelry, Inc. will recover $2,000, the loss less the amount in excess of the $10,000

display limitation.

RPCPA, AICPA. CMA & CIA EXAMINATION QUESTIONS Page 26 of 55

Page 27: T04 - Risks & Cost of Capital

MANAGEMENT ADVISORY SERVICES RISKS

C. Jewelry, Inc. will recover the full $4,000 since the warranty will be construed as a mere representation.

D. Jewelry, Inc. will recover the full $4,000 since attaching the application to the policy is insufficient to make it a part thereof. AICPA 1181 L-55

RPCPA, AICPA. CMA & CIA EXAMINATION QUESTIONS Page 27 of 55

Page 28: T04 - Risks & Cost of Capital

MANAGEMENT ADVISORY SERVICES RISKS

RPCPA, AICPA. CMA & CIA EXAMINATION QUESTIONS Page 28 of 55

Page 29: T04 - Risks & Cost of Capital

MANAGEMENT ADVISORY SERVICES RISKS

Investment RiskFinancial RiskOptimal Capital StructureCost of CapitalDebt Capital vs. Equity CapitalImputed Costs vs. Explicit CostsCost of Debt CapitalMarginal Cost of DebtCost of Equity CapitalMarginal Cost of CapitalOptimal Cost of CapitalDividend Growth ModelCapital Asset Pricing Model (CAPM)LeverageProbability & Statistics

ANSWER EXPLANATIONS

RPCPA, AICPA. CMA & CIA EXAMINATION QUESTIONS Page 29 of 55

Page 30: T04 - Risks & Cost of Capital

MANAGEMENT ADVISORY SERVICES RISKS

13 . Answer (A) is correct. Business risk is the risk of fluctuations in earnings before interest 7. Corporate risk Answer: e Diff: M8 . Answer (B) is correct. Total risk is the risk of a single asset, whereas market risk is its risk if it is held in a large portfolio of diversified securities. Total risk therefore includes diversifiable and

undiversifiable risk. Answer (A) is incorrect because nonmarket risk is diversifiable and undiversifiable risk. Answer (C) is incorrect because portfolio risk is diversifiable and undiversifiable risk. Answer (D) is incorrect because market risk is diversifiable and undiversifiable risk.

9 . Answer (D) is correct. The greater the standard deviation of returns, the greater the risk is for an asset. The expected return can vary anywhere between the large standard deviation of returns, creating the risk that the actual return is significantly low in the range of the standard deviation of returns. Answer (A) is incorrect because an asset with high risk will have a high expected return to compensate for the additional risk. Answer (B) is incorrect because an asset with high risk will have a higher price than an asset with low risk due to the high expected return. Answer (C) is incorrect because an asset with high risk will have a constant expected return, not an increasing expected return.

10 . REQUIRED: The circumstance in which overall company risk will be lowest.DISCUSSION: (A) A common general definition is that risk is an investment with an unknown outcome but a known probability distribution of returns (a known mean and standard deviation). An increase in the standard deviation (variability) of returns is synonymous with an increase in the riskiness of a project. Risk is also increased when the project’s returns are positively (directly) correlated with other investments in the firm’s portfolio, i.e, risk increases when returns on all projects rise or fall together. Consequently, the overall risk is decreased with projects have low variability and are negatively correlated (the diversification effect).Answers (B), (C), and (D) are incorrect because overall risk is decreased when projects have low variability and are negatively correlated.

11 . Answer (D) is correct. An asset is liquid if it can be converted to cash on short notice. Liquidity (marketability) risk is the risk that assets cannot be sold at a reasonable price on short notice. If an asset is not liquid, investors will require a higher return than for a liquid asset. The difference is the liquidity premium. Answer (A) is incorrect because default risk is the risk that a borrower will not pay the interest or principal on a loan. Answer (B) is incorrect because interest-rate risk is the risk to which investors are exposed because of changing interest rates. Answer (C) is incorrect because purchasing-power risk is the risk that inflation will reduce the purchasing power of a given sum of money.

12 . Answer (D) is correct. Liquidity risk is the possibility that an asset cannot be sold on short notice for its market value. If an asset must be sold at a high discount, it is said to have a substantial amount of liquidity risk. Answer (A) is incorrect because interest rate risk is caused by fluctuations in the value of an asset as interest rates change. Its components are price risk and reinvestment-rate risk. Answer (B) is incorrect because purchasing power risk is the risk that a general rise in the price level (inflation) will reduce what can be purchased with a fixed sum of money. Answer (C) is incorrect because financial risk is the risk borne by shareholders, in excess of basic business risk, that arises from use of financial leverage (issuance of fixed income securities, i.e., debt and preferred stock.

RPCPA, AICPA. CMA & CIA EXAMINATION QUESTIONS Page 30 of 55

Page 31: T04 - Risks & Cost of Capital

MANAGEMENT ADVISORY SERVICES RISKS

and taxes or in operating income when the firm uses no debt. It depends on factors such as demand variability, sales price variability, input price variability, and the amount of operating leverage. Financial leverage affects financial risk and is not a factor affecting business risk. Answer (B) is incorrect because sales price variability is a factor affecting business risk. Answer (C) is incorrect because demand variability is a factor affecting business risk. Answer (D) is incorrect because input price variability is a factor affecting business risk.

14 . Answer (A) is correct. Business risk is the risk of fluctuations in earnings before interest and taxes or in operating income when the firm uses no debt. It is the risk inherent in its operations that excludes financial risk, which is the risk to the shareholders from the use of financial leverage. Business risk depends on factors such as demand variability, sales price variability, input price variability, and amount of operating leverage. Answer (B) is incorrect because business risk depends on such factors as amount of operating leverage. Answer (C) is incorrect because business risk depends on such factors as demand variability. Answer (D) is incorrect because business risk depends on such factors as fluctuations in suppliers' prices.

15 . Statements a and b are correct; therefore, statement d is the correct choice. High uncertainty about future sales implies high business risk. The extent to which a firm’s costs are fixed affects business risk. Financial leverage does not affect the firm’s business risk. Financial leverage shifts the business risk between classes of ownership, that is, debt and equity.

16 . REQUIRED: The set of ratios that characterizes the firm with the greatest financial risk.DISCUSSION: (C) A firm with high risk will have a higher debt-to-equity ratio than a low risk firm, a lower interest coverage, and a volatile return on equity. A higher debt-to-equity ratio poses a greater risk of insolvency. Debtholders must be paid regardless of whether the firm is profitable. Low interest coverage means that the margin of safety of earnings before interest and taxes is small. A volatile return on equity signifies that earnings are unpredictable. Lack of predictability increases risk.Answer (A) are incorrect because high interest coverage suggests lower risk. Answer (B) is incorrect because a low debt-to-equity ratio indicates a low level of risk. Answer (D) is incorrect because a low debt-to-equity ratio, high interest coverage, and a stable return on equity suggests lower risk.

17 . Answer (D) is correct. When amounts to be paid or received are denominated in a foreign currency, exchange rate fluctuations may result in exchange gains or losses. For example, if a U.S. firm has a receivable fixed in terms of units of a foreign currency, a decline in the value of that currency relative to the U.S. dollar results in a foreign exchange loss. Answer (A) is incorrect because sovereign or expropriation risk is the risk that the sovereign country in which the assets backing an investment are located will seize the assets without adequate compensation. Answer (B) is incorrect because sovereign or expropriation risk is the risk that the sovereign country in which the assets backing an investment are located will seize the assets without adequate compensation. Answer (C) is incorrect because the beta value in the capital asset pricing model for a multinational firm is the systematic risk of a given multinational firm relative to that of the market as a whole.

18 . Answer (C) is correct. Exchange-rate risk is the risk that a foreign currency transaction will be negatively exposed to fluctuations in exchange rates. Because O & B Company sells goods to German customers and records accounts receivable denominated in deutsche marks, O & B Company is exposed to exchange-rate risk.

RPCPA, AICPA. CMA & CIA EXAMINATION QUESTIONS Page 31 of 55

Page 32: T04 - Risks & Cost of Capital

MANAGEMENT ADVISORY SERVICES RISKS

19 . Answer (A) is correct. Extension of credit in a foreign currency would result in receiving payment in less valuable dollars if the foreign currency became less valuable. Thus one would not want to encourage granting trade credit in a foreign country when the country's currency is expected to lose value. Answer (B) is incorrect because investing monetary assets into nonmonetary assets is advantageous when the monetary unit is going to lose value. Answer (C) is incorrect because it is advantageous to become a debtor when the monetary unit is losing value. Answer (D) is incorrect because it is advantageous to become a debtor when the monetary unit is losing value.

20 . Answer (D) is correct. A firm may buy or sell forward exchange contracts to cover liabilities or receivables, respectively, denominated in a foreign currency. Any gain or loss on the foreign receivables or payables because of changes in exchange rates is offset by the loss or gain on the foreign exchange contract. Answer (A) is incorrect because in general, a firm should attempt to achieve a net monetary debtor position in countries with currencies expected to depreciate. Answer (B) is incorrect because a net monetary creditor position should be achieved in countries with strengthening currencies. Answer (C) is incorrect because a firm may seek to minimize its exchange-rate risk by diversification. If it has transactions in both strong and weak currencies, the effects of changes in rates may be offsetting.

21 . Answer (B) is correct. Political risk is the risk that a foreign government may act in a way that will reduce the value of the company's investment. Political risk may be reduced by making foreign operations dependent on the domestic parent for technology, markets, and supplies. Answer (A) is incorrect because political risk may be reduced by entering into a joint venture with a company from the host country rather than from a foreign country. Answer (C) is incorrect because refusing to pay higher wages and higher taxes will only increase political risk. Answer (D) is incorrect because political risk may be reduced by financing with local capital, rather than foreign capital.

22 . Answer (C) is correct. A feasible portfolio that offers the highest expected return for a given risk or the least risk for a given expected return is called an efficient portfolio. Answer (A) is incorrect because an optimal portfolio is a portfolio selected from the efficient set of portfolios because it is tangent to the investor's highest indifference curve. Answer (B) is incorrect because a desirable portfolio is a nonsense term. Answer (D) is incorrect because an effective portfolio is a nonsense term.

23 . Answer (D) is correct. An investor wants to maximize expected return and minimize risk when choosing a portfolio. A feasible portfolio that offers the highest expected return for a given risk or the least risk for a given expected return is an efficient portfolio. A portfolio that is selected from the efficient set of portfolios because it is tangent to the investor's highest indifference curve is the optimal portfolio. Answer (A) is incorrect because a portfolio is efficient if it offers the highest return for a given risk or the least risk for a given return. Answer (B) is incorrect because the optimal portfolio is tangent to the investor's highest indifference curve. Thus, it is the efficient portfolio with the highest utility. Answer (C) is incorrect because the optimal portfolio is efficient as well as feasible.

24 . Answer (A) is correct. A common general definition is that risk is an investment with an unknown outcome but a known probability distribution of returns (a known mean and standard deviation). An increase in the standard deviation (variability) of returns is synonymous with an increase in the riskiness of a project. Risk is also increased when the project's returns are positively (directly) correlated with other investments in the company's portfolio; that is, risk increases when returns on all projects rise or fall together. Consequently, the overall risk is decreased when projects have low variability and are negatively correlated (the diversification effect).

RPCPA, AICPA. CMA & CIA EXAMINATION QUESTIONS Page 32 of 55

Page 33: T04 - Risks & Cost of Capital

MANAGEMENT ADVISORY SERVICES RISKS

25 . Answer (B) is correct. Business units may be treated as elements of an investment portfolio. A portfolio should be efficient in balancing the risk with the rate of return on the portfolio. The expected rate of return of a portfolio is the weighted average of the expected returns of the individual assets in the portfolio. The variability (risk) of a portfolio's return is determined by the correlation of the returns of individual portfolio assets. To the extent the returns are not perfectly positively correlated, variability is decreased. Thus, business units should be selected that increase returns and diversify and reduce risk. Answer (A) is incorrect because Michael Porter's model of competitive strategies has two variables: competitive advantage and competitive scope. The strategy adopted depends on whether the advantage sought is based on lower cost or product differentiation, and on whether the scope is broad or narrow. Answer (C) is incorrect because scenario development is a qualitative forecasting method that involves preparing conceptual scenarios of future events, given carefully defined assumptions. It entails writing multiple alternative but equally likely descriptions of future states. A longitudinal scenario indicates how the current circumstances may develop, and a cross-sectional scenario describes possible future states at a designated time. Answer (D) is incorrect because situational analysis is a method of determining an organization's direction by systematically matching its strengths and weaknesses with its environmental opportunities and threats (referred to as a SWOT analysis).

26 . Market, or systematic, or nondiversifiable, risk is present in a diversified portfolio; the unsystematic risk has been eliminated. 27 . The risk remaining in diversified portfolios is systematic risk; thus, portfolio returns are commensurate with systematic risk.

28 . A and b quantify the extra return investors demand to bear portfolio risk.

29 . Answer (A) is correct. The general principle is that risk and return are directly correlated. U.S. Treasury securities are backed by the full faith and credit of the federal government and are therefore the least risky form of investment. However, their return is correspondingly lower. Corporate first mortgage bonds are less risky than income bonds or stock because they are secured by specific property. In the event of default, the bondholders can have the property sold to satisfy their claims. Holders of first mortgages have rights paramount to those of any other parties, such as holders of second mortgages. Income bonds pay interest only in the event the corporation earns income. Thus, holders of income bonds have less risk than stockholders because meeting the condition makes payment of interest mandatory. Preferred stockholders receive dividends only if they are declared, and the directors usually have complete discretion in this matter. Also, stockholders have claims junior to those of debtholders if the enterprise is liquidated. Answer (B) is incorrect because the proper listing is mortgage bonds, subordinated debentures, income bonds, and preferred stock. Debentures are unsecured debt instruments. Their holders have enforceable claims against the issuer even if no income is earned or dividends declared. Answer (C) is incorrect because the proper listing is first mortgage bonds, second mortgage bonds, income bonds, and common stock. The second mortgage bonds are secured, albeit junior, claims. Answer (D) is incorrect because the proper listing is mortgage bonds, debentures, preferred stock, and common stock. Holders of common stock cannot receive dividends unless the holders of preferred stock receive the stipulated periodic percentage return, in addition to any averages if the preferred stock is cumulative.

30 . REQUIRED: The least risky security from the viewpoint of the investor.DISCUSSION: (A) A mortgage bond is secured with specific fixed assets, usually real property. Thus, under the rights enumerated in the bond indenture, creditors will be able to receive payments from liquidation of the property in case of default. In a bankruptcy proceeding, these amounts are paid before any transfers are made to other creditors, including those preferences. Hence, mortgage bonds

RPCPA, AICPA. CMA & CIA EXAMINATION QUESTIONS Page 33 of 55

Page 34: T04 - Risks & Cost of Capital

MANAGEMENT ADVISORY SERVICES RISKS

Answer (A) is incorrect because liquidity risk is the possibility that an asset cannot be sold on 31 . Answer (D) is correct. The standard deviation is a measure of the degree of compactness of the values in a population. It is a measure of dispersion. The standard deviation is found by taking the

square root of the quotient of the sum of the squared deviations from the mean, divided by the number of items in the population. Corn has a mean return of 20% and a standard deviation of 15%. Hence, it is not as risky as Must, which has a standard deviation of 9% relative to a mean of only 10%. The coefficient of variation (standard deviation ÷ expected return) is much higher for Must (0.9 ÷ .10 = .9) than for Corn (.15 ÷ .20 = .75). Answer (A) is incorrect because the existence of a higher return is not necessarily indicative of high risk. Answer (B) is incorrect because the higher standard deviation must be viewed relative to the mean of the population. Answer (C) is incorrect because Must does not have the higher standard deviation.

32 . Answer (A) is correct. The required rate of return on equity capital in the capital asset pricing model is the risk-free rate (determined by government securities) plus the product of the market risk premium times the beta coefficient (beta measures the firm's risk). The market risk premium is the amount above the risk-free rate that will induce investment in the market. The beta coefficient of an individual stock is the correlation between the volatility (price variation) of the stock market and that of the price of the individual stock. Answer (B) is incorrect because the coefficient of variation is the standard deviation of an investment's returns divided by the mean return. Answer (C) is incorrect because the standard deviation is a measure of the variability of an investment's returns. Answer (D) is incorrect because the beta coefficient measures the sensitivity of the investment's returns to market volatility.

33 . Answer (C) is correct. The CAPM measures the tradeoff between investment return and risk. The return is the risk-free return plus a risk premium. Thus, the CAPM theory analyzes portfolios in terms of their rate of return and risk, and the effect of adding additional securities to the portfolio. Answer (A) is incorrect because CAPM is a portfolio (not single security) theory. Answer (B) is incorrect because the CAPM is a general theory that is not specific to any security trading strategies. Answer (D) is incorrect because the CAPM applies to investment securities, not fixed assets.

34 . With a diversified portfolio, the only risk remaining is market, or systematic, risk. This is the only risk that influences return according to the CAPM.

35 . Answer (C) is correct. The risk of concern to investors who supply capital to a diversified company is market risk. Beta or market risk is the risk that cannot be eliminated by diversification. This type of risk is defined from the perspective of an investor who views the investment as just one part of a diversified portfolio. It is the average risk of the firm's investment projects calculated as the weighted average of project betas. Answer (A) is incorrect because, in project risk assessment, the firm attempts to assess the level of risk for a given project by considering, for a given project, either pure play betas or accounting betas (beta values determined from accounting data rather than from regression of a company's stock returns on a stock market index). Neither is a sufficient means for assessing the overall risk of a diversified firm. Answer (B) is incorrect because, in the pure play method of project risk assessment, the firm attempts to estimate beta for a single product company in the same line of business as the project being evaluated. Answer (D) is incorrect because, in project risk assessment, the firm attempts to assess the level of risk for a given project by considering, for a given project, either pure play betas or accounting betas (beta values determined from accounting data rather than from regression of a company's stock returns on a stock market index). Neither is a sufficient means for assessing the overall risk of a diversified firm.

RPCPA, AICPA. CMA & CIA EXAMINATION QUESTIONS Page 34 of 55

Page 35: T04 - Risks & Cost of Capital

MANAGEMENT ADVISORY SERVICES RISKS

36 . Answer (A) is correct. There is a positive relationship between a firm's beta value and the discount rate applied to cash flows. Thus, a decrease in beta value will reduce the discount rate. Answer (B) is incorrect because the discount rate will be reduced, not increased. Answer (C) is incorrect because the discount rate will be reduced. Answer (D) is incorrect because a zero discount rate incorrectly suggests that future cash flows do not need to be discounted for evaluation purposes.

37 . REQUIRED: The riskier stock.DISCUSSION: (E) The standard deviation is a measure of the degree of compactness of the values in a population. It is a measure of dispersion. The standard deviation is found by taking the square root of the quotient of the sum of the squared deviations from the mean, divided by the number of items in the population. In other words, the standard deviation describes how far from the mean the various elements of the population are. Cornhusker has a mean return of 20% and a standard deviation of 15%. Hence, it is not as risky as Mustang, which has a standard deviation of 9% relative to a mean of only 10%. The coefficient of variation (standard deviation ÷ expected return) is must higher for Mustang (0.09 ÷ 0.10 = 0.9) than for Cornhusker (0.15 ÷ 0.20 = 0.75).Answer (A) is incorrect because the existence of a higher return is not necessarily indicative of high risk. Answer (B) is incorrect because is incorrect because the higher standard deviation must be viewed relative to the mean of the population; the absolute level of the standard deviation is meaningless without a knowledge of the mean. Answer (C) is incorrect because Mustang does not have the higher standard deviation. Answer (D) is incorrect because the absolute level of Mustang’s rate of return is not indicative of risk.

38 . Answer (D) is correct. An important measurement used in portfolio analysis is the covariance. It measures the volatility of returns together with their correlation with the returns of other securities. For two stocks X and Y, if k is the expected return, ki is a given outcome, and pi is its probability, the covariance of X and Y is

Answer (A) is incorrect because the variance and standard deviation are calculated for a single investment. The standard deviation gives an exact value for the tightness of the distribution and the riskiness of the investment. The standard deviation (å) is the square root of the variance. Answer (B) is incorrect because the variance and standard deviation are calculated for a single investment. The standard deviation gives an exact value for the tightness of the distribution and the riskiness of the investment. The standard deviation (å) is the square root of the variance. Answer (C) is incorrect because the coefficient of variation is calculated for a single investment. The coefficient of variation is useful when the rates of return and standard deviations of two investments differ. It measures the risk per unit of return because it divides the standard deviation by the expected return.

39 . Answer (B) is correct. The covariance measures the volatility of returns together with their correlation with the returns of other securities. It is calculated with the following equation:

The covariance of two stocks is the same regardless of which stock is compared to the other. Answer (A) is incorrect because the covariance of stock B with stock A is the same as the covariance of stock A with stock B. Answer (C) is incorrect because the covariance of stock B with stock A is the same as the covariance of stock A with stock B. Answer (D) is incorrect because the covariance of stock B with stock A is the same as the covariance of stock A with stock B.

RPCPA, AICPA. CMA & CIA EXAMINATION QUESTIONS Page 35 of 55

Page 36: T04 - Risks & Cost of Capital

MANAGEMENT ADVISORY SERVICES RISKS

41 . Answer (A) is correct. The expected rate of return of a portfolio is the weighted average of the expected returns of the individual assets in the portfolio. Answer (B) is incorrect because the interest rate for the safest possible investment is directly used in the capital asset pricing model. Answer (C) is incorrect because the expected rate of return on the market portfolio is the one systematic risk factor in the model. Answer (D) is incorrect because the standard deviation of the market returns is used to compute the beta coefficients for each security held in the investment portfolio and the beta coefficient for the market portfolio.

42 . Answer (D) is correct. The correlation coefficient (r) measures the degree to which any two variables are related. It ranges from -1.0 to 1.0. Perfect positive correlation (1.0) means that the two variables always move together. Perfect negative correlation (-1.0) means that the two variables always move inversely to one another. A neutral correlation, or no correlation, is 0.0. Skewed is a nonsense concept in this context. Answer (A) is incorrect because returns on two stocks can be correlated in any value that falls within the range of -1.0 to 1.0. Answer (B) is incorrect because returns on two stocks can be correlated in any value that falls within the range of -1.0 to 1.0. Answer (C) is incorrect because returns on two stocks can be correlated in any value that falls within the range of -1.0 to 1.0.

43 . Answer (D) is correct. Maturity matching, or equalizing the life of an asset and the debt instrument used to finance that asset, is a hedging approach. The basic concept is that the company has the entire life of the asset to recover the amount invested before having to pay the lender. Answer (A) is incorrect because working capital management is short-term asset management. Answer (B) is incorrect because return maximization is more aggressive than maturity matching. It entails using the lowest cost forms of financing. Answer (C) is incorrect because financial leverage is the relationship between debt and equity financing.

44 . Answer (A) is correct. Swaps are contracts to hedge risk by exchanging cash flows. In an interest-rate swap, one firm exchanges its fixed interest and principal payments for a series of payments based on a floating rate. If a firm has debt with fixed charges, but its revenues fluctuate with interest rates, it may prefer to swap for cash outflows based on a floating rate. The advantage is that revenues and the amounts of debt service will then move in the same direction, and interest-rate risk will be reduced. A currency swap is an exchange of an obligation to pay out cash flows denominated in one currency for an obligation to pay in another. For example, a U.S. firm with revenues in francs has to pay suppliers and workers in dollars, not francs. To minimize exchange-rate risk, it might agree to exchange francs for dollars held by a firm that needs francs. The exchange rate will be an average of the rates expected over the life of the agreement.Answer (B) is incorrect because contracts to hedge risk by exchanging cash flows include interest-rate swaps and currency swaps. Answer (C) is incorrect because contracts to hedge risk by exchanging cash flows include interest-rate swaps and currency swaps. Answer (D) is incorrect because contracts to hedge risk by exchanging cash flows include interest-rate swaps and currency swaps.

45 . Answer (C) is correct. Derivatives, including options and futures, are contracts between the parties who contract. Unlike stocks and bonds, they are not claims on business assets. A futures contract is entered into as either a speculation or a hedge. Speculation involves the assumption of risk in the hope of gaining from price movements. Hedging is the process of using offsetting commitments to minimize or avoid the impact of adverse price movements. Answer (A) is incorrect because hedging decreases risk by using offsetting commitments that avoid the impact of adverse price movements. Answer (B) is incorrect because speculation involves the assumption of risk in the hope of gaining from price movements. Answer (D) is incorrect because speculating increases risk while hedging offsets risk.

RPCPA, AICPA. CMA & CIA EXAMINATION QUESTIONS Page 36 of 55

Page 37: T04 - Risks & Cost of Capital

MANAGEMENT ADVISORY SERVICES RISKS

47 . Answer (C) is correct. The goal of duration hedging is not to equate the duration of assets and the duration of liabilities but for the following relationship to apply: (Value of assets) X (Duration of assets)(Value of liabilities) X (Duration of liabilities)The firm is immunized against interest-rate risk when the total price change for assets equals the total price change for liabilities. Answer (A) is incorrect because if duration increases, the volatility of the price of a debt instrument. Answer (B) is incorrect because the goal is to equate the total price change for assets and the total price change for liabilities. Answer (D) is incorrect because duration is lower if the nominal rate on a debt instrument is higher.

48 . Answer (C) is correct. A forward contract is an executory contract in which the parties involved agree to the terms of a purchase and a sale, but performance is deferred. Accordingly, a forward contract involves a commitment today to purchase a product on a specific future date at a price determined today. Answer (A) is incorrect because the price of a future contract is determined on the day of commitment, not some time in the future. Answer (B) is incorrect because performance is deferred in a future contract, and the price of the product is not necessarily its present price. The price can be any price determined on the day of commitment. Answer (D) is incorrect because a forward contract is a firm commitment to purchase a product. It is not based on a contingency. Also, a forward contract does not involve an exercise price (exercise price is in an option contract).

49 . Answer (B) is correct. Interest rate futures contracts involve risk-free bonds, such as U.S. Treasury bonds. When interest rates decrease over the period of a forward contract, the value of the bonds and the forward contract increase. Answer (A) is incorrect because the value of the forward contract will increase when interest rates decrease. Answer (C) is incorrect because the value of the forward contract will increase when interest rates decrease. Answer (D) is incorrect because any decline in interest rates increases the value of the bonds.

50 . Answer (D) is correct. A distinguishing feature of futures contracts is that their prices are marked to market every day at the close of the day. Thus, the market price is posted at the close of business each day. A mark-to-market provision minimizes a futures contract's chance of default because profits and losses on the contracts must be received or paid each day through a clearinghouse. Answer (A) is incorrect because both a forward contract and a futures contract are executory. Answer (B) is incorrect because a futures contract may be speculative. Answer (C) is incorrect because a futures contract is for delivery during a given month.

51 . Answer (B) is correct. A change in prices can be minimized or avoided by hedging. Hedging is the process of using offsetting commitments to minimize or avoid the impact of adverse price movements. The automobile company desires to stabilize the price of steel so that its cost to the company will not rise and cut into profits. Accordingly, the automobile company uses the futures market to create a long hedge, which is a futures contract that is purchased to protect against price increases. Answer (A) is incorrect because a short hedge is a futures contract that is sold to protect against price declines. The automobile company wishes to protect itself against price increases. Answer (C) is incorrect because the automobile company needs to purchase futures in order to protect itself from loss, not sell futures. Selling futures protects against price declines. Answer (D) is incorrect because it is the definition of a short hedge, which is used for avoiding price declines. The automobile company wants to protect itself against price increases.

52 . Answer (D) is correct. In an interest rate swap, two companies exchange their debt servicing obligations. Such a transaction arises because the companies (called the counter parties) wish to limit interest rate risk. If a company with a variable-rate loan desires the certainty of a fixed-rate loan, it may swap with a company that prefers the floating rate. The actual exchange of funds during the

RPCPA, AICPA. CMA & CIA EXAMINATION QUESTIONS Page 37 of 55

Page 38: T04 - Risks & Cost of Capital

MANAGEMENT ADVISORY SERVICES RISKS

53 . Answer (A) is correct. Swaps are contracts to hedge risk by exchanging cash flows. In an interest-rate swap, one firm exchanges its fixed interest and principal payments for a series of payments based on a floating rate. If a firm has debt with fixed charges, but its revenues fluctuate with interest rates, it may prefer to swap for cash outflows based on a floating rate. The advantage is that revenues and the amounts of debt service will then move in the same direction, and interest-rate risk will be reduced. A currency swap is an exchange of an obligation to pay out cash flows denominated in one currency for an obligation to pay in another. For example, a U.S. firm with revenues in francs has to pay suppliers and workers in dollars, not francs. To minimize exchange-rate risk, it might agree to exchange francs for dollars held by a firm that needs francs. The exchange rate will be an average of the rates expected over the life of the agreement. Answer (B) is incorrect because contracts to hedge risk by exchanging cash flows include interest-rate swaps and currency swaps. Answer (C) is incorrect because contracts to hedge risk by exchanging cash flows include interest-rate swaps and currency swaps. Answer (D) is incorrect because contracts to hedge risk by exchanging cash flows include interest-rate swaps and currency swaps.

54 . Answer (A) is correct. Under sensitivity analysis, forecasts of many calculated NPVs under various assumptions are compared to see how sensitive the NPV of a possible project is to the change of a single input variable. In this instance, the single unit variable being changed is the price of a table. Answer (B) is incorrect because simulation analysis requires the use of a computer to generate many examples of results based upon various assumptions. Answer (C) is incorrect because the informal method calculates NPVs at the firm's discount rate (k) and individually reviews the possible projects. Answer (D) is incorrect because certainty equivalent adjustments are not frequently used since decision makers are not familiar with its concept. It forces the decision maker to specify at what point the firm is indifferent to the choice between a certain sum of money and the expected value of a risky sum.

55 . Answer (A) is correct. Simulation analysis considers both the sensitivity of the NPV when variables are changed and the range of values of the variables that are changed. Answer (B) is incorrect because The Capital Asset Pricing Model is derived from the use of portfolio theory. Answer (C) is incorrect because sensitivity analysis only considers the sensitivity of the NPV when variables are changed, not the range of values of the variables that are changed. Answer (D) is incorrect because certainty equivalent adjustments does not involve calculating NPVs.

56 . Answer (D) is correct. Risk analysis attempts to measure the likelihood of the variability of future returns from the proposed investment. Risk can be incorporated into capital budgeting decisions in a number of ways, one of which is to use a hurdle rate higher than the firm's cost of capital, that is, a risk-adjusted discount rate. This technique adjusts the interest rate used for discounting upward as an investment becomes riskier. The expected flow from the investment must be relatively larger, or the increased discount rate will generate a negative net present value, and the proposed acquisition will be rejected. Accordingly, the IRR (the rate at which the NPV is zero) for a rejected investment may exceed the cost of capital when the risk-adjusted rate is higher than the IRR. Conversely, the IRR for an accepted investment may be less than the cost of capital when the risk-adjusted rate is less than the IRR. In this case, the investment presumably has very little risk. Furthermore, risk-adjusted rates may also reflect the differing degrees of risk, not only among investments, but by the same investments undertaken by different organizational subunits. Answer (A) is incorrect because discount rates may vary with the project or with the subunit of the organization. Answer (B) is incorrect because the company may accept some projects with IRRs less than the cost of capital, or reject some project with IRRs greater than the cost of capital. Answer (C) is incorrect because the company may accept some projects with IRRs less than the cost of capital, or reject some project with IRRs greater than the cost of capital.

57 . ks = 10% + (16% - 10%)1.5 = 10% + 9% = 19%.Expected return = 21%. 21% - Risk adjustment 1% = 20%.

RPCPA, AICPA. CMA & CIA EXAMINATION QUESTIONS Page 38 of 55

Page 39: T04 - Risks & Cost of Capital

MANAGEMENT ADVISORY SERVICES RISKS

Risk-adjusted return = 20% > ks = 19%.59

?. Calculate the beta of the firm, and use to calculate project beta:ks = 0.16 = 0.10 + (0.05)bFirm. bFirm = 1.2. bProject = (bFirm)1.5 (bProject is 50% greater than current bFirm)bProject = (1.2)1.5 = 1.8.Calculate required return on project, kProject, and compare to IRR.Project: kProject = 0.10 + (0.05)1.8 = 0.19 = 19%. IRR = 0.18 = 18%.

Since the required return is one percentage point greater than the expected IRR, the firm should not accept the new project.

60 . Answer (C) is correct. The theory underlying the cost of capital is based primarily on the cost of long-term funds and the acquisition of new funds. The reason is that long-term funds are used to finance long-term investments. For an investment alternative to be viable, the return on the investment must be greater than the cost of the funds used. The objective in short-term borrowing is different. Short-term loans are used for working capital, not to finance long-term investments. Answer (A) is incorrect because the concern is with the cost of new funds; the cost of old funds is a sunk cost and of no relevance for decision-making purposes. Answer (B) is incorrect because the cost of short-term funds is not usually a concern for investment purposes. Answer (D) is incorrect because there is less concern with the cost of old funds or short-term funds.

61 . Answer (C) is correct. The cost of capital is the minimum, not maximum, rate of return that must be earned on new investments so as not to dilute shareholder interest. If new investments have a rate of return less than the cost of capital, a loss will be incurred on those investments. Thus, the cost of capital must be the minimum rate of return. See SMA 4A, Cost of Capital. Answer (A) is incorrect because cost of capital is used to make capital investment decisions so that each investment returns more than the cost of capital. Answer (B) is incorrect because the cost of maintaining working capital is based on the cost of capital. Answer (D) is incorrect because the performance of individual investments, investment managers, and others can be related to the cost of capital.

62 . Answer (D) is correct. The U-shaped curve indicates that the cost of capital is quite high when the debt-to-equity ratio is quite low. As debt increases, the cost of capital declines as long as the cost of debt is less than that of equity. Eventually, the decline in the cost of capital levels off because the cost of debt ultimately rises as more debt is used. Additional increases in debt (relative to equity) will then increase the cost of capital. The implication is that some debt is present in the optimal capital structure because the cost of capital initially declines when debt is added. However, a point is reached at which debt becomes excessive and the cost of capital begins to rise. Answer (A) is incorrect because the composition of the capital structure affects the cost of capital since the components have different costs. Answer (B) is incorrect because the cost of debt does not remain constant as financial leverage increases. Eventually, that cost also increases. Answer (C) is incorrect because the initial decline in the U-shaped graph indicates that the financial markets reward moderate levels of debt.

63 . Answer (A) is correct. The overall cost of capital is the rate of return on a firm's assets that exactly covers the costs associated with the funds employed. It is the weighted average of the various debt and equity components.

RPCPA, AICPA. CMA & CIA EXAMINATION QUESTIONS Page 39 of 55

Page 40: T04 - Risks & Cost of Capital

MANAGEMENT ADVISORY SERVICES RISKS

64 . Answer (A) is correct. An imputed cost is one that has to be estimated. It is a cost that exists but is not specifically stated and is the result of a process designed to recognize economic reality. An imputed cost may not require a dollar outlay formally recognized by the accounting system, but it is relevant to the decision-making process. For example, the stated interest on a bank loan is not an imputed cost because it is specifically stated and requires a dollar outlay. But the cost of using retained earnings as a source of capital is unstated and has to be imputed. Answer (B) is incorrect because the cost of obsolete assets should be written off. Answer (C) is incorrect because understated depreciation results in unstated costs. Answer (D) is incorrect because the lower-than-market return on a loan is an imputed cost of the products.

65 . REQUIRED: The implicit cost of financing.DISCUSSION: (B) Debt capital often appears to have a lower cost than equity because the implicit costs are not obvious. The implicit costs are attributable to the increased risk created by the additional debt burden. Thus, as the debt-to-equity ratio increases, the cost of both debt and equity will increase given the increased risk to both shareholders and creditors from a higher degree of leverage. An explanation based on the marginal cost of capital and the marginal efficiency of investment leads to the same conclusion. Lower cost capital sources are used first. Additional projects must then be undertaken with funds from higher cost sources. Similarly, risk is increased because the most profitable investments are made initially, leaving the less profitable investments for the future.Answer (A) is incorrect because both debt and equity sources increase in cost as leverage increases. Answer (C) is incorrect because equity costs decline as leverage decreases. Answer (D) is incorrect because the weighted-average cost of capital will increase with increased leverage.

66 . Retained earnings are just another form of equity. When the company has retained earnings, they can do one of two things--reinvest them or pay them out as dividends. If they reinvest the earnings, they need to earn a return that is at least as high as the ks of the stock. Otherwise, investors would be happier getting the dividends and investing them in something that will get them ks. Therefore, statement a is false. Some of the preferred stock dividends are excluded from taxation when another company owns them. It makes no tax difference to the company that pays the dividends, since dividends come out of after-tax dollars. Therefore, statement b is false. Interest payments are tax deductible. Therefore, statement c is true.

67 . Answer (B) is correct. The cost of debt capital is simply the debt interest rate times (1 - the firm's tax rate). Thus, if the tax rate is 40%, the effective cost of debt capital is 60% times the interest rate because the interest is tax deductible. See SMA 4A, Cost of Capital. Answer (A) is incorrect because it ignores the tax deductibility of interest payments. Answer (C) is incorrect because it ignores the tax deductibility of interest payments. Answer (D) is incorrect because the capital asset pricing model is one means of determining the cost of common equity.

68 . Answer (D) is correct. The after-tax cost of debt is the cost of debt times the quantity one minus the tax rate. For example, the after-tax cost of a 10% bond is 7% [10% x (1 - 30%)] if the tax rate is 30%. Answer (A) is incorrect because the after-tax cost of debt is the cost of debt times the quantity one minus the tax rate. Answer (B) is incorrect because the after-tax cost of debt is the cost of debt times the quantity one minus the tax rate. Answer (C) is incorrect because the cost of debt times the marginal tax rate equals the tax savings from issuing debt.

69 . Answer (D) is correct. Because interest is deductible for tax purposes, the actual cost of debt capital is the net effect of the interest payment and the offsetting tax deduction. The actual cost of debt equals the interest rate times one minus the marginal tax rate. Thus, if a firm with an 8% market rate is in a 40% tax bracket, the net cost of the debt capital is 4.8% [8% x (1 - .4)].

RPCPA, AICPA. CMA & CIA EXAMINATION QUESTIONS Page 40 of 55

Page 41: T04 - Risks & Cost of Capital

MANAGEMENT ADVISORY SERVICES RISKS

short notice for its market value. Answer (B) is incorrect because business risk is the risk of 70 . Answer (C) is correct. As a larger proportion of an entity's capital is provided by debt, the debt becomes riskier and more expensive, i.e., requires a higher interest rate.

Answer (A) is incorrect because the diversity decreases, not increases, risk. Answer (B) is incorrect because $50 million is minuscule in the debt markets. Answer (D) is incorrect because the combination alternative maintains the same debt/equity mixture, which would not warrant a rate increase in the cost of debt or equity.

71 . Answer (C) is correct. The excess of the price over the face value is a premium. A premium is paid because the coupon rate on the bond is greater than the market rate of interest. In other words, because the bond is paying a higher rate than other similar bonds, its price is bid up by investors. Answer (A) is incorrect because, if a bond sells at a premium, the market rate of interest is less than the coupon rate. Answer (B) is incorrect because a bond sells at a discount when the price is less than the face amount. Answer (D) is incorrect because a bond sells at a discount when the price is less than the face amount.

72 . REQUIRED: The definition of the marginal cost of debt.DISCUSSION: (C) The marginal cost of debt is calculated as the cost of new debt times one minus the marginal tax rate, or Kd (1 - T). This d expression equals Kd - KdT .Answer (A) is incorrect because the marginal cost of debt financing is the interest rate on new debt minus the firm's marginal tax rate multiplied by the interest rate. Answer (B) is incorrect because the marginal cost of debt financing is the interest rate on new debt minus the firm's marginal tax rate multiplied by the interest rate. Moreover, the marginal or incremental cost of debt to the firm is based on the cost of newly issued debt, not on the cost of outstanding debt. Answer (D) is incorrect because the marginal cost of debt financing is the interest rate on new debt minus the firm's marginal tax rate multiplied by the interest rate. Moreover, the marginal or incremental cost of debt to the firm is based on the cost of newly issued debt, not on the cost of outstanding debt.

73 . The debt cost used to calculate a firm’s WACC is kd(1 - T). If kd remains constant but T increases, then the term (1 - T) decreases and the value of the entire equation, kd(1 - T), decreases. Statement b is false; if a company’s stock price increases, and all else remains constant, then the dividend yield decreases and ks decreases. This can be seen from the equation ks = D1/P0 + g. Statement c is false for the same reason. The cost of issuing new common stock is ke = D1/[P0(1 - F)] + g. If P0 increases but there’s no change in the flotation cost, ke will decrease.

74 . Answer (B) is correct. Providers of equity capital are exposed to more risk than are lenders because the firm is not obligated to pay them a return. Also, in case of liquidation, creditors are paid before equity investors. Thus, equity financing is more expensive than debt because equity investors require a higher return to compensate for the greater risk assumed. Answer (A) is incorrect because the obligation to repay at a specific maturity date reduces the risk to investors and thus the required return. Answer (C) is incorrect because the demand for equity capital is directly related to its greater cost to the issuer. Answer (D) is incorrect because dividends are based on managerial discretion and may rarely change; interest rates, however, fluctuate daily based upon market conditions.

75 . Statement a is correct. Most preferred stock is owned by corporations which receive a 70 percent exclusion of dividends. Consequently, the before-tax coupons on preferred stock are often lower than the before-tax coupons on debt, despite the fact that preferred stock is riskier than debt. All the other statements are false.

76 . Answer (B) is correct. The dividend growth model is used to calculate the price of stock.RPCPA, AICPA. CMA & CIA EXAMINATION QUESTIONS Page 41 of 55

Page 42: T04 - Risks & Cost of Capital

MANAGEMENT ADVISORY SERVICES RISKS

fluctuations in earnings before interest and taxes or in operating income when the firm uses no debt. Answer (D) is incorrect because price risk is a component of interest-rate risk.

77 . REQUIRED: The item resulting in a higher market value for common shares.DISCUSSION: (A) The dividend growth model is used to calculate the cost of equity. The simplified formula is

R is the required rate of return, D is the next dividend, P is the stock’s price, and G is the growth rate in earnings per share. The equation is also used to determine the stock price.

Thus, when investors have a lower required return on equity, the denominator is smaller, which translates in to a higher market value.Answer (B) is incorrect because, if investors expect lower dividend growth, the market value of common shares will decrease. Answers (C) and (D) are incorrect because the expected holding periods of investors are not related to the market value of the common shares.

78 . Answer (B) is correct. Newly issued or external common equity is more costly than retained earnings. The company incurs issuance costs when raising new, outside funds. Answer (A) is incorrect because the cost of retained earnings is the rate of return stockholders require on equity capital the firm obtains by retaining earnings. The opportunity cost of retained funds will be positive. Answer (C) is incorrect because retained earnings will always be less costly than external equity financing. Earnings retention does not require the payment of issuance costs. Answer (D) is incorrect because retained earnings will always be less costly than external equity financing. Earnings retention does not require the payment of issuance costs.

79 . Statement a is correct; the other statements are false. Preferred stock dividends are not tax deductible; therefore, the cost of preferred stock is only kp. The risk premium in the bond-yield-plus-risk premium approach would be added to the firm’s cost of debt, not the risk-free rate. Preferred stock also has flotation costs.

80 . REQUIRED: The effect of a higher dividend-payout ratio.DISCUSSION: (A) the higher the dividend-payout ratio, the sooner retained earnings are exhausted and the company must seek external financing. Assuming the same investments are undertaken, the result is a higher marginal cost of capital because lower-cost capital sources will be used up earlier.Answer (B) is incorrect because the marginal cost of capital is higher. Answers (C) and (D) are incorrect because the existence of investment opportunities is unrelated to the dividend payout.

81 . Answer (D) is correct. The marginal cost of capital is the cost of the next dollar of capital. The marginal cost continually increases because the lower cost sources of funds are used first. The marginal cost represents a weighted average of both debt and equity capital. Answer (A) is incorrect because, if the cost of capital were the same as the rate of return on equity (which is usually higher than that of debt capital), there would be no incentive to invest. Answer (B) is incorrect because the marginal cost of capital is affected by the degree of debt in the firm's capital structure. Financial risk plays a role in the returns desired by investors. Answer (C) is incorrect because the rate of return used for capital budgeting purposes should be at least as high as the marginal cost of capital.

RPCPA, AICPA. CMA & CIA EXAMINATION QUESTIONS Page 42 of 55

Page 43: T04 - Risks & Cost of Capital

MANAGEMENT ADVISORY SERVICES RISKS

82 . Answer (B) is correct. The measure of the value of an individual stock is dependent entirely upon the stream of future cash flows that it will produce. To determine the stock's current value, these cash flows should be discounted to time zero (now) to obtain the stream's present value. Stocks primarily provide cash flows to investors via dividends (including share repurchases and liquidating dividends) and capital gain (loss) at the time of sale. Once the stream of cash flows has been discounted over a significant number of periods, it is easy to see that the dividend stream, not the capital gain (loss) in the final period, drives the value of the stock in question. Of course, all firms do not pay a dividend. Common financial theory, however, states that it is the intention of every firm to pay a dividend to shareholders at some time in the future, once the firm feels it is strong enough to do so and still support future operations. After all, it is the primary goal of a firm's management to maximize shareholder wealth. Although many factors should be considered when purchasing a security, the primary consideration for a value-seeking investor is the future cash flow stream. Answer (A) is incorrect because book value is a measure of the stock's worth on a company's accounting records. Answer (C) is incorrect because the beta coefficient is a measure of how volatile the price movements of a stock are relative to the market as a benchmark. Answer (D) is incorrect because standard deviation is a measure of risk. While risk is a consideration for the investor, one of the fundamental concepts in finance is that there is (should be) a trade-off between risk and return, and as long as risk is compensated for, it is not a primary consideration.

83 . Answer (D) is correct. The dividend growth model is used to calculate the cost of equity. The simplified formula is

R = + G

R is the required rate of return, D1 is the next dividend, PO is the stock's price, and G is the growth rate in earnings per share. The equation is also used to determine the stock price.

Answer (A) is incorrect because the model uses the growth rate in dividends and the investor's required rate of return. Answer (B) is incorrect because the model uses the growth rate in dividends and the investor's required rate of return. Answer (C) is incorrect because the model uses the growth rate in dividends and the investor's required rate of return.

84 . Answer (D) is correct. The Gordon (dividend) growth model requires three elements to estimate the cost of equity capital. These are the dividends per share, the expected growth rate in dividends per share, and the market price of the stock. Basically, the cost of equity capital can be computed as the dividend yield (dividends ÷ price) plus the growth rate. Answer (A) is incorrect because book value per share is not a consideration in computing the cost of equity capital. Answer (B) is incorrect because current dividends, not current earnings, per share are a requirement for the formula. Answer (C) is incorrect because book value per share is not a consideration in computing the cost of equity capital. Also, current dividends, not current earnings, per share are a requirement for the formula.

85 . The CAPM assumes that investors are price-takers with the same single holding period and that there are no taxes or transaction costs.

86 . The SML is a measure of expected return per unit of risk, where risk is defined as beta (systematic risk).

RPCPA, AICPA. CMA & CIA EXAMINATION QUESTIONS Page 43 of 55

Page 44: T04 - Risks & Cost of Capital

MANAGEMENT ADVISORY SERVICES RISKS

88 . Both the Capital Market Line and the Security Market Line depict risk/return relationships. However, the risk measure for the CML is standard deviation and the risk measure for the SML is beta (thus c is not true; the other statements are true).

89 . In equilibrium, the marginal price of risk for a risky security must be equal to the marginal price of risk for the market. If not, investors will buy or sell the security until they are equal.

90 . An underpriced security will have a higher expected return than the SML would predict; therefore it will plot above the SML.

91 . Answer (A) is correct. The capital asset pricing model adds the risk-free rate to the product of the market risk premium and the beta coefficient. The market risk premium is the amount above the risk-free rate (approximated by the U.S. treasury bond yield) that must be paid to induce investment in the market. The beta coefficient of an individual stock is the correlation between the price volatility of the stock market as a whole and the price volatility of the individual stock. Answer (B) is incorrect because the price-earnings ratio is not a component of the model. Answer (C) is incorrect because the price-earnings ratio is not a component of the model. Answer (D) is incorrect because the dividend payout ratio is not a component of the model.

92 . The expected rate of return on any security is equal to the risk free rate plus the systematic risk of the security (beta) times the market risk premium, E(rM - Rf).

93 . The tangency point between the capital market line and the indifference curve is the optimal portfolio for a particular investor. 94 . A zero alpha results when the security is in equilibrium (fairly priced for the level of risk).

95 . A security with a positive alpha is one that is expected to yield an abnormal rate of return, based on the perceived risk of the security, and thus is underpriced.

96 . Answer (B) is correct. The required rate of return on equity capital in the Capital Asset Pricing Model is the risk-free rate (determined by government securities), plus the product of the market risk premium times the beta coefficient (beta measures the firm's risk). The market risk premium is the amount above the risk-free rate that will induce investment in the market. The beta coefficient of an individual stock is the correlation between the volatility (price variation) of the stock market and that of the price of the individual stock. For example, if an individual stock goes up 15% and the market only 10%, beta is 1.5. Answer (A) is incorrect because the coefficient of variation compares risk with expected return (standard deviation ÷ expected return). Answer (C) is incorrect because standard deviation measures dispersion (risk) of project returns. Answer (D) is incorrect because expected return does not describe risk.

RPCPA, AICPA. CMA & CIA EXAMINATION QUESTIONS Page 44 of 55

Page 45: T04 - Risks & Cost of Capital

MANAGEMENT ADVISORY SERVICES RISKS

98 . Answer (C) is correct. The relevant risk of a security is its contribution to the portfolio's risk. It is the risk that cannot be eliminated through diversification and is synonymous with market risk and systematic risk. The relevant risk results from factors, such as inflation, recession, and high interest rates, that affect all stocks. Answer (A) is incorrect because company-specific risk can be eliminated through portfolio diversification. Answer (B) is incorrect because diversifiable risk can be eliminated through portfolio diversification. Answer (D) is incorrect because only the systematic component of total risk is relevant to security valuation.

99 . Beta is a measure of how a security's return covaries with the market returns, normalized by the market variance.

100 . By definition, the beta of the market portfolio is 1.

101 . Once, a portfolio is diversified, the only risk remaining is systematic risk, which is measured by beta.

102 . E(RS) = rf + 0(RM - rf) = rf.

103 . The market rewards systematic risk, which is measured by beta, and thus, the risk premium on a stock or portfolio varies directly with beta.

104 . The risk premium on the market portfolio is proportional to the average degree of risk aversion of the investor population and the risk of the market portfolio measured by its variance. 105. Calculate the required return, ks, and use to calculate the WACC:

ks = 10% + 1.38(5%) = 16.9%.WACC = 0.5(12.0%)(0.6) + 0.5(16.9%) = 12.05%.Compare expected project return, , to WACC:

But = IRR = 13.0%.Accept the project since IRRProject > WACC: 13.0% > 12.05%.

106 . Answer (B) is correct. To estimate the required rate of return on equity, the capital asset pricing model (CAPM) adds the risk-free rate (determined by government securities) to the product of the beta coefficient (a measure of the firm's risk) and the difference between the market return and the risk-free rate. Below is the basic equilibrium equation for the CAPM.

R = RF + ß (RM - RF )Thus, given a beta of 1.2, R is 11% [5% + 1.2 (10% - 5%)]. At a beta of 1.5, R is 12.5% [5% + 1.5 (10% - 5%)]. Answer (A) is incorrect because 3% equals the market return times the increase in the beta. Answer (C) is incorrect because the company's required rate of return is affected by a change in the company's beta coefficient. Answer (D) is incorrect because the change results in an increase of the company's required rate of return, not a decrease.

RPCPA, AICPA. CMA & CIA EXAMINATION QUESTIONS Page 45 of 55

Page 46: T04 - Risks & Cost of Capital

MANAGEMENT ADVISORY SERVICES RISKS

107 . 11% = 5% + 1.5(9% - 5%) = 11.0%; therefore, the security is fairly priced.

108 . 12% < 7% + 1.3(15% - 7%) = 17.40%; therefore, stock is overpriced and should be shorted.

109 . A:12% - [5% + 1.2(9% - 5%)] = 2.2%.

110 . Because corporations can exclude dividends for tax purposes, preferred stock often has a before-tax market return that is less than the issuing company’s before-tax cost of debt. Then, if the issuer’s tax rate is zero, its component cost of preferred would be less than its after-tax cost of debt.

111 . Answer (B) is correct. The cost of capital of a firm is the current, weighted average, after-tax cost of the firm's various financing components. Historical costs are irrelevant. Answer (A) is incorrect because the cost of capital is a weighted average for all sources of capital. Answer (C) is incorrect because costs are considered after taxes. For example, the deductibility of interest must be considered. Answer (D) is incorrect because the time value of money should be incorporated into the calculations.

112 . Answer (D) is correct. A weighted average of the costs of all financing sources should be used, with the weights determined by the usual financing proportions. The terms of any financing raised at the time of initiating a particular project do not represent the cost of capital for the firm. When a firm achieves its optimal capital structure, the weighted-average cost of capital is minimized. Answer (A) is incorrect because the cost of capital is a composite, or weighted average, of all financing sources in their usual proportions. The cost of capital should also be calculated on an after-tax basis. Answer (B) is incorrect because the cost of capital is a composite, or weighted average, of all financing sources in their usual proportions. The cost of capital should also be calculated on an after-tax basis. Answer (C) is incorrect because the cost of capital is a composite, or weighted average, of all financing sources in their usual proportions. The cost of capital should also be calculated on an after-tax basis.

113 . Answer (A) is correct. The cost of capital of a firm is the current, weighted-average, after-tax cost of the firm's various financing components. Historical costs are irrelevant. Answer (B) is incorrect because costs are considered after taxes. For example, the deductibility of interest must be considered. Answer (C) is incorrect because the time value of money should be incorporated into the calculations. Answer (D) is incorrect because the cost of capital is a weighted average for all sources of capital.

114 . If a firm paid no income taxes, its cost of debt would not be adjusted downward, hence the component cost of debt would be higher than if T were greater than 0. With a higher component cost of debt, the WACC would be increased. Of course, the company would have higher earnings, and its cash flows from a given project would be high, so the higher WACC would not impede its investments, that is, its capital budget would be larger than if it were taxed.

115 . Statements a and c are both correct; therefore, statement d is the correct choice. Statement a recites the definition of the weighted average cost of capital. Statement c is correct because kd = kRF + LP + MRP + DRP while ks = kRF + (kM - kRF)b. If kRF increases then the values for kd and ks will increase.

RPCPA, AICPA. CMA & CIA EXAMINATION QUESTIONS Page 46 of 55

Page 47: T04 - Risks & Cost of Capital

MANAGEMENT ADVISORY SERVICES RISKS

Answer (B) is incorrect because uncorrelated investments are more risky than negatively 116 . Statement a is correct; the other statements are false. Statement b is incorrect; WACC is an average of debt and equity financing. Since debt financing is cheaper and is adjusted downward for taxes,

it should, when averaged with equity, cause the WACC to be less than the cost of equity financing. Statement c is incorrect; WACC is calculated on an after-tax basis. Statement d is incorrect; the WACC is based on marginal, not embedded, costs. Statement e is incorrect; the cost of issuing new common stock is greater than the cost of retained earnings.

117 . WACC measures the marginal after-tax cost of capital; therefore, statement a is false. The after-tax cost of debt financing is less than the after-tax cost of equity financing; therefore, statement b is false. The correct choice is statement c.

118 . The preferred stock dividend is not tax deductible like the interest payment on debt. Therefore, there is no tax benefit from preferred stock. Statement a is true. Retained earnings are equity, and equity will have a higher cost than debt. Therefore, statement b is false. If the beta increases, investors will require a higher rate of return to hold or buy the stock. Therefore, the cost of equity will go up, and statement c is true. Because statements a and c are true, the correct choice is statement e.

119 . Statements a and b are true. Statement c is false. The cost of new stock is higher than the cost of retained earnings because there are flotation costs associated with issuing new stock. Since statements a and b are true the correct choice is statement d.

120 . Statement c is the correct choice. A tax rate increase would lead to a decrease in the after-tax cost of debt and, consequently, the firm’s WACC would decrease.

121 . Statement a is correct; the other statements are false. If RPM decreases, the cost of equity will be reduced. Answers b through e will all increase the company’s WACC.

122

?. Calculate the required return, ks, and compare to the expected return, . = 7%.

ks = kRF + (kM - kRF)b = 0.07 + (0.10 - 0.07)0.5 = 0.085 = 8.5%.ks > ; 8.5% > 7.0%; reject the investment.

123 . Calculate the required return, ks, and use to calculate the WACC:ks = 10% + 1.38(5%) = 16.9%.WACC = 0.5(12.0%)(0.6) + 0.5(16.9%) = 12.05%.Compare expected project return, , to WACC:

But = 13.0%.

Accept the project since > WACC: 13.0% > 12.05%.RPCPA, AICPA. CMA & CIA EXAMINATION QUESTIONS Page 47 of 55

Page 48: T04 - Risks & Cost of Capital

MANAGEMENT ADVISORY SERVICES RISKS

124

?. Calculate the after-tax component cost of debt as 10%(1 - 0.3) = 7%. If the company has earnings of $100,000 and pays out 50% or $50,000 in dividends, then it will retain earnings of $50,000. The retained earnings breakpoint is $50,000/0.4 = $125,000. Since it will require financing in excess of $125,000 to undertake any of the alternatives, we can conclude the firm must issue new equity. Therefore, the pertinent component cost of equity is the cost of new equity. Calculate the expected dividend per share (note this is D1) as $50,000/10,000 = $5. Thus, the cost of new equity is $5/[($35(1 - 0.12)] + 6% = 22.23%. Jackson’s WACC is 7%(0.6) + 22.23%(0.4) = 13.09%. Only the return on Project A exceeds the WACC, so only Project A will be undertaken.

125 . Answer (C) is correct. The degree of operating leverage is the percentage change in EBIT resulting from a given percentage change in sales. Thus, it causes EBIT to be more sensitive to changes in sales. Answer (A) is incorrect because the term credit is not applicable to leverage. Answer (B) is incorrect because financial leverage refers to the sensitivity of EPS to EBIT. Answer (D) is incorrect because the term intrinsic is not applicable to leverage.

126 . Answer (A) is correct. Operating leverage is based on the degree to which fixed costs are used in production. Firms may increase fixed costs, such as by automation, to reduce variable costs. The result is a greater degree of operating leverage (DOL), which is the percentage change in earnings before interest and taxes (net operating profit) divided by the percentage change in unit sales. It can also be determined from the following formula, given that Q is quantity of units sold, P is unit price, V is unit variable cost, and F is fixed cost:

Answer (B) is incorrect because the degree of financial leverage equals the percentage change in EPS divided by the percentage change in net operating profit. Answer (C) is incorrect because the breakeven point is the sales volume at which total revenue equals total costs. Answer (D) is incorrect because the degree of total (combined) leverage equals the percentage change in EPS divided by the percentage change in sales.

127 . Answer (C) is correct. The degree of operating leverage (DOL) is a measure of the change in earnings available to common stockholders associated with a given change in operating earnings. It is calculated, for a particular level of sales, as:

Answer (A) is incorrect because DOL varies with the sales level. Answer (B) is incorrect because the degree of financial leverage is a measure of the change in earnings available to common stockholders associated with a given change in operating earnings. Answer (D) is incorrect because the degree of total leverage is the multiple of the degree of operating leverage and the degree of financial leverage. Other things being equal, DOL is higher if the degree of total leverage is higher.

RPCPA, AICPA. CMA & CIA EXAMINATION QUESTIONS Page 48 of 55

Page 49: T04 - Risks & Cost of Capital

MANAGEMENT ADVISORY SERVICES RISKS

130 . Answer (B) is correct. A purchase of treasury stock involves a decrease in assets (usually cash) and a corresponding decrease in shareholders' equity. Thus, equity is reduced and the debt-to-equity ratio and financial leverage increase. Answer (A) is incorrect because assets decrease when treasury stock is purchased. Answer (C) is incorrect because a firm's interest coverage ratio is unaffected. Earnings, interest expense, and taxes will all be the same regardless of the transaction. Answer (D) is incorrect because the purchase of treasury stock is antidilutive; the same earnings will be spread over fewer shares. Some firms purchase treasury stock for this reason.

131 . REQUIRED: The effect of increasing the DFL.DISCUSSION: (D) The DFL equals the percentage change in earnings available to common shareholders divided by the percentage change in net operating income. When the DFL rises, fixed interest charges and the riskiness of the firm rise. As a result, the variability of the returns to equity holders will increase. In other words, the standard deviation of returns on the equity of the company rises.Answer (A) is incorrect because an increase in the DFL increases the riskiness of the firm’s stock. Thus, beta rises. Beta is a measure of the volatility of a firm’s stock price relative to the average stock. Answers (B) and (C) are incorrect because systematic risk, also known as market risk, is unrelated to the DFL. Systematic risk is not specific to a company. It is the risk associated with a company’s stock that cannot be diversified because it arises from factors that affect all stocks.

132 . Answer (C) is correct. Financial leverage is the use of borrowed money to earn money for the benefit of shareholders. The expectation is that investment earnings will be greater than the interest paid on the borrowed funds. Increasing debt (such as bonds) increases financial leverage. Answer (A) is incorrect because the issuance of common stock does not increase financial leverage. No increase in borrowed capital and fixed interest charges occurs when equity is issued. Answer (B) is incorrect because a decrease in the dividend payout ratio would result in increased owners' equity (retained earnings), and would not increase debt capital and financial leverage. Answer (D) is incorrect because using debt, not equity, funds to finance new investments increases financial leverage.

133 . Answer (C) is correct. Earnings per share is less volatile in less highly leveraged firms. Lower fixed costs result in less variable earnings when sales fluctuate. Answer (A) is incorrect because higher leverage is associated with higher, not lower, EPS when sales exceed the breakeven point. Answer (B) is incorrect because earnings per share is more volatile in more highly leveraged firms. Answer (D) is incorrect because less leverage is associated with lower, not higher, EPS when sales exceed the breakeven point.

134 . Answer (B) is correct. Financial leverage is the amount of the fixed cost of capital, principally debt, in a firm's capital structure relative to its operating profit. Leverage creates financial risk and is directly related to the cost of capital. Because the company is retiring bonds, the total debt is decreased. Given that the amount of debt and leverage are directly related, a decrease in the amount of debt results in a decrease in financial leverage. Answer (A) is incorrect because the bond retirement decreases the debt-equity ratio. Answer (C) is incorrect because the total assets will decrease (assets will be used to retire the debt, resulting in an increased asset turnover ratio (net sales ÷ average total assets). Answer (D) is incorrect because the interest expense avoided will increase net profit and the return on shareholders' equity.

135 . Statement a is true; a higher EPS does not always mean that the stock price will increase. Statement b is false; a lower WACC will mean a higher stock price. Statement c is false; EPS can increase just because shares outstanding decline.

RPCPA, AICPA. CMA & CIA EXAMINATION QUESTIONS Page 49 of 55

Page 50: T04 - Risks & Cost of Capital

MANAGEMENT ADVISORY SERVICES RISKS

136 . Statement a is incorrect because BEP = EBIT/Total assets. The extent to which the firm uses debt financing does not effect EBIT or total assets. Statement b is incorrect because firms with a high percentage of fixed costs have a high degree of operating leverage by definition.

137 . BEP = EBIT/TA. Since they both have the same total assets and the same BEP, then EBIT must be the same for both companies. If A has a higher debt ratio and higher interest expense than B, and they both have the same EBIT and tax rate, then A must have a lower NI than B. Therefore, statement a is true. If A has a lower NI than B but both have the same total assets, then A’s ROA (NI/TA) must be lower than B’s ROA. Therefore, statement b is true. If both companies have the same total assets but A’s debt ratio is higher than B’s, then A’s equity must be lower (since Total assets = Total debt + Total equity). If A has less equity, and a lower NI than B, it is not possible to judge which company’s ROE (NI/EQ) is higher.

138 . BEP = EBIT/TA. If both firms have the same BEP ratio and same total assets, then they must have the same EBIT. Since Firm U has no debt in its capital structure, Firm U will have higher net income than Firm L because U has no interest expense and L does. The TIE ratio is EBIT/Int. If the two companies have the same EBIT, the one with the lower interest expense (Firm U), will have a higher TIE. Therefore, statement a is false. Firms L and U have the same EBIT, but Firm L has a higher interest expense, so its net income will be lower than Firm U. Since ROA is equal to NI/TA, and the two firms have the same total assets, Firm L will have a lower ROA than Firm U. Therefore, statement b is true. Leverage will increase ROE if BEP > k d. Since BEP is 20 percent and kd is 8 percent, leverage will increase Firm L’s ROE. Therefore, statement c is false.

139 . Statement a is false; A’s net income is lower than B’s due to higher interest expense, but its assets are equal to B’s, so A’s ROA must be lower than B’s ROA. Statement b is false; A has the same EBIT as B, but higher interest payments than B; therefore, A’s TIE is lower than B’s. Statement c is correct.

140 . Answer (B) is correct. Leverage is the amount of the fixed cost of capital, principally debt, in a firm's capital structure relative to its operating income. It is also defined as the ratio of debt to total assets or debt to capital. Leverage, by definition, creates financial risk, which relates directly to the question of the cost of capital. The more leverage, the higher the financial risk, and the higher the cost of debt capital. The degree of financial leverage (DFL) is the percentage change in earnings available to common stockholders that is associated with a given percentage change in net operating income (earnings before interest and taxes). The more financial leverage employed, the greater the DFL, and the riskier the firm. Whenever the return on assets is greater than the cost of debt, additional leverage is favorable. An increase in the equity component of the capital structure, however, decreases financial leverage. Operating leverage is a related concept based on the degree that fixed costs are used in the production process. A company with a high percentage of fixed costs is riskier than a firm in the same industry that relies more on variable costs to produce. The degree of operating leverage (DOL) is the percentage change in net operating income that is associated with a given percentage change in sales. When fixed assets increase, operating leverage also increases. Answer (A) is incorrect because financial leverage would decrease and operating leverage would increase. Answer (C) is incorrect because financial leverage would decrease and operating leverage would increase. Answer (D) is incorrect because financial leverage would decrease and operating leverage would increase.

141 . Answer (D) is correct. Insurance is a method of spreading losses that arise from risks to which many persons are subject. Loss is an unanticipated diminution in economic value as opposed to normal depreciation. Risk is uncertainty about the occurrence or the amount of loss. For example, buildings are subject to the risk of loss by fire. If the owners all pay small fees (premiums) for insurance coverage, every participant bears part of the loss instead of a few bearing all the loss.

RPCPA, AICPA. CMA & CIA EXAMINATION QUESTIONS Page 50 of 55

Page 51: T04 - Risks & Cost of Capital

MANAGEMENT ADVISORY SERVICES RISKS

143 . Answer (A) is correct. An insurance contract is very similar to any other contract. However, an additional requirement is that the insured must have an insurable interest. Answer (B) is incorrect because there is no general requirement that an insurance contract be written. Oral binders are given every day in the insurance business. Answer (C) is incorrect because consideration is needed for the initial formation of an insurance contract. The premium may be paid immediately, or a promise to pay is required. Answer (D) is incorrect because the insurance company can also commit a breach by refusing to pay the proceeds of the policy upon the occurrence of the event.

144 . Answer (A) is correct. Life insurance is usually purchased to protect against the cessation of income needed for support of the family and to shield them from the decedent's debts. Answer (B) is incorrect because life insurance is customarily long-term, if not for life. Answer (C) is incorrect because, unlike other forms of insurance, life insurance does not attempt to reimburse for the actual amount of a loss since loss of life is not measurable. Life insurance is intended to replace economic benefits lost by a person's death. Answer (D) is incorrect because, except for term policies, life insurance differs from other kinds of coverage in providing cash value.

145 . Answer (D) is correct. Term life insurance provides protection for a specified period. Premiums are level throughout the period. When the term ends, the insured receives no payment. Term insurance may be renewable (possibly at higher premiums) or convertible to another form. It is the cheapest kind of life insurance. Answer (A) is incorrect because whole life furnishes lifetime insurance protection with a cash surrender value. Answer (B) is incorrect because an endowment policy provides life insurance protection for its duration. A cash payment is made (the policy endows) at the end of the term. Premiums for endowment policies are higher than for whole life insurance. Answer (C) is incorrect because a straight life or ordinary policy is whole life insurance with level premiums payable for life.

146 . Answer (B) is correct. When one person insures the life of another, the policyholder must have an insurable interest in the insured. That interest is found among persons who have a close family relationship or expect to suffer substantial economic loss from the death. Business entities are thus permitted to insure key people in their organizations whose death might have an adverse effect on profits.Answer (A) is incorrect because a corporation has an insurable interest only in its key employees, i.e., those whose death would cause loss to the firm. Answer (C) is incorrect because the required loss need not be so great as to cause cessation of business. Answer (D) because one need not be an owner or an officer to be insurable as a key person.

147 . Answer (A) is correct. Fire insurance is the most standardized kind of insurance. Following the lead of New York, almost all states have enacted a standard policy either by legislative or administrative action. Answer (B) is incorrect because, given that fire insurance is usually written for a 1- to 3-year period, an incontestability clause is not necessary. Such a clause bars insurer defenses after a period specified by law or the policy. Answer (C) is incorrect because a policy may state a definite value of the insured property or simply a maximum amount of coverage that is not conclusive as to valuation when loss occurs. A policy may thus be valued or open (unvalued). A pro rata clause is often included (but not required) which requires the loss to be shared pro rata when there is more than one insurer. Answer (D) is incorrect because the insurable interest merely requires the person, e.g., mortgagee, bailee, etc., to suffer a loss if the event insured against occurs.

148 . Answer (D) is correct. If a premium is not received by the due date, the policyholder has a grace period under state law, usually a month or 31 days, in which to pay. After the grace period, the cash surrender value is not forfeited but can be withdrawn or used to buy a paid-up policy.

RPCPA, AICPA. CMA & CIA EXAMINATION QUESTIONS Page 51 of 55

Page 52: T04 - Risks & Cost of Capital

MANAGEMENT ADVISORY SERVICES RISKS

149 . Answer (B) is correct. Ordinarily, smoke, water, or other damage caused by hostile, but not friendly, fires will be indemnified under a fire insurance policy. Hostile fires are those ignited in places where they are not meant to be. A friendly fire is one that burns where it is intended to burn, such as a fireplace or furnace. For example, if a friendly fire is kept within its usual container, damage caused by smoke from it will not be reimbursed. Answer (A) is incorrect because fire insurance usually indemnifies for loss caused by hostile, but not friendly, fires. Answer (C) is incorrect because fire insurance usually indemnifies for loss caused by hostile, but not friendly, fires. Answer (D) is incorrect because fire insurance usually indemnifies for loss caused by hostile, but not friendly, fires.

150 . Answer (C) is correct. Arson, fraud, or another intentional act of the insured calculated to cause the damage insured against will preclude recovery. The parties to an insurance contract have an implied duty not to bring about the very event that is the subject matter of the policy. Answer (A) is incorrect because agency rules do not apply; i.e., the intentional act of an agent will not be imputed to the insured under the doctrine of respondeat superior. Arson by an agent without the actual knowledge or conspiracy of the insured will not preclude recovery. Answer (B) is incorrect because arson is compensable unless intended by the insured. Answer (D) is incorrect because negligence without fraud will not prevent recovery by an insured who has acted in good faith.

151 . Answer (D) is correct. Fire insurance generally protects the insured from damage to the insured property as a result of fire. It does not cover the insured for causing a fire on someone else's property. Answer (A) is incorrect because malpractice insurance is a special form of liability insurance protecting professionals from lawsuits by third parties for negligence. Answer (B) is incorrect because homeowners insurance generally contains a liability section in the event guests are injured on the premises. Answer (C) is incorrect because a primary purpose of automobile insurance is to protect the owner or driver from liability in the event (s)he is responsible for damage to another person or property.

152 . Answer (C) is correct. Co-insurance is a method of sharing risk between the insurer and the insured. A co-insurance clause typically provides that, if the policyholder insures his/her property for at least a stated percentage (usually 80%) of its actual cash value, any loss will be paid in full up to the face amount of the policy. Thus, a co-insurance clause is used by many property and casualty insurers to encourage policyholders to insure commercial property for an amount that is near to its full replacement cost. Answer (A) is incorrect because, if a policyholder does not insure the property for an amount close enough to its full replacement cost, the policyholder must bear a proportionate part of any partial loss. Answer (B) is incorrect because the co-insurance requirement applies only to partial losses. Total losses result in recovery of the face amount of the policy. Answer (D) is incorrect because a co-insurance clause encourages policyholders to insure commercial property for an amount that is near to its full replacement cost.

153 . Answer (A) is correct. Conditions precedent, called warranties, are part of the property insurance policy. Breach of a warranty precludes recovery and results in a forfeiture. Since the law disfavors forfeitures, courts construe questions of interpretation favorably to the insured and against the insurer that drafted the policy. However, if the parties expressly agree that certain statements are warranties, then a court would recognize them as warranties. Since Jewelry warranted never to display more than $10,000 of jewelry, the breach of warranty prevents recovery. Answer (B) is incorrect because Jewelry will recover nothing; it is not entitled to $2,000. Answer (C) is incorrect because the warranty will not be construed as a mere representation; the intention of the parties clearly was to make it a warranty. Answer (D) is incorrect because attachment of the application to the policy is usually sufficient to make it a part thereof. It may also be incorporated into the policy by reference to it.

RPCPA, AICPA. CMA & CIA EXAMINATION QUESTIONS Page 52 of 55

Page 53: T04 - Risks & Cost of Capital

MANAGEMENT ADVISORY SERVICES RISKS

Answer (A) is incorrect because risk avoidance and loss control do not transfer risk of loss. Answer (B) is incorrect because an insurable interest is merely a potential for economic loss if an event occurs. Answer (C) is incorrect because there must be an insurable interest, which is basically potential for loss if an event occurs. Gambling occurs when only a bet is at risk.

128 . Answer (A) is correct. The degree of operating leverage (DOL) is equal to the percentage change in net operating income, that is, earnings before interest and taxes (EBIT) divided by the percentage change in sales. Because interest expenses are not included in EBIT, the DOL is not affected by a change in interest expense. Answer (B) is incorrect because variable cost per unit affects EBIT and therefore the DOL. Answer (C) is incorrect because quantity of units sold affects EBIT and therefore the DOL. Answer (D) is incorrect because fixed costs affect EBIT and therefore the DOL.

correlated investments. Answer (C) is incorrect because correlated investments are very risky. Answer (D) is incorrect because correlated investments are very risky.

are less risky than the others listed.Answer (B) is incorrect because a debenture is long-term debt that is not secured (collateralized) by specific property. Subordinated debentures have a claim on the debtor’s assets that may be satisfied only after senior debt has been paid in full. Debentures of either kind are therefore more risky than mortgage bonds. Answer (C) is incorrect because an income bond pays interest only if the debtor earns it. Such bonds are also more risky than secured debt. Answer (D) is incorrect because unsecured debt is riskier than a mortgage bond.

40 . Answer (C) is correct. An indifference curve represents combinations of portfolios having equal utility to the investor. Given that risk and returns are plotted on the horizontal and vertical axes, respectively, and that the investor is risk averse, the curve has an increasingly positive slope. As risk increases, the additional required return per unit of additional risk also increases. The steeper the slope of an indifference curve, the more risk averse an investor is. The higher

the curve, the greater is the investor's level of utility. Answer (A) is incorrect because the slope is less steep if the investor is less risk averse. The increase in the required additional return per unit of additional risk is lower. Answer (B) is incorrect because the higher the curve, the greater is the investor's level of utility. Moreover, the slope is positive, not negative. Answer (D) is incorrect because the slope is increasingly positive.

46 . Answer (B) is correct. A put option is the right to sell stock at a given price within a certain period. If the market price falls, the put option may allow the sale of stock at a price above market, and the profit of the option holder will be the difference between the price stated in the put option and the market price, minus the cost of the option, commissions, and taxes. The company that issues the stock has nothing to do with put (and call) options. Answer (A) is incorrect because a call option is the right to purchase shares at a given price within a specified period. Answer (C) is incorrect because selling a put option could force the company to purchase additional stock if the option is exercised. Answer (D) is incorrect because a warrant gives the holder a right to purchase stock from the issuer at a given price (it is usually distributed along with debt).

agreement is in the form of a net payment from the party owing the greater amount for the period. Interest rate swaps are a form of off-balance-sheet financing because only the original borrowing, not the swap agreement, appears on the balance sheet.Answer (A) is incorrect because an interest rate swap is not a sale of a right to borrow at a preferential rate. Answer (B) is incorrect because the parties to an interest rate swap deal only with each other. Answer (C) is incorrect because an interest rate swap is an exchange of interest obligations, not an offsetting of interest income and interest expense.

58

?. Calculate the beta of the firm, and use to calculate project beta:ks = 0.16 = 0.10 + (0.05)bFirm. bFirm = 1.2.bProject = (bFirm)1.5. (bProject is 50% greater than current bFirm)bProject = (1.2)1.5 = 1.8.

RPCPA, AICPA. CMA & CIA EXAMINATION QUESTIONS Page 53 of 55

Page 54: T04 - Risks & Cost of Capital

MANAGEMENT ADVISORY SERVICES RISKS

Calculate required return on project, kProject, and compare to expected return:Project: kProject = 0.10 + (0.05)1.8 = 0.19 = 19%. Expected return = 0.18 = 18%. Since the required return is one percentage point greater than the expected return, the firm should not accept the new project.

Answer (B) is incorrect because the cost of capital is based on what a company pays for its capital, not the return earned on the capital employed. Answer (C) is incorrect because the overall cost of capital is the minimum rate a firm must earn on all investments to cover capital costs. Answer (D) is incorrect because the overall cost of capital is based on both debt and equity components.

Answer (A) is incorrect because the tax deduction always causes the market yield rate to be higher than the cost of debt capital. Answer (B) is incorrect because additional debt may or may not be issued more cheaply than earlier debt, depending upon the interest rates in the market place. Answer (C) is incorrect because the cost of debt is less than the yield rate given that bond interest is tax deductible.

If: PO = current price D1 = next dividend R = required rate of return G = EPS growth rateAssuming that D1 and G remain constant, an increase in R resulting from an increase in the nominal interest rate will cause PO to decrease. Answer (A) is incorrect because a higher interest rate raises the required return of investors, which results in a lower stock price. Answer (C) is incorrect because a higher interest rate raises the required return of investors, which results in a lower stock price. Answer (D) is incorrect because a higher interest rate raises the required return of investors, which results in a lower stock price.

87 . The SML is a measure of expected return-beta (the CML is a measure of expected return-standard deviation of market returns). The SML provides the expected return-beta relationship for "fairly priced" securities; thus if a portfolio manager selects securities that are underpriced and produces a portfolio with a positive alpha, this portfolio manager would receive a positive evaluation.

97 . Answer (A) is correct. The word beta is derived from the regression equation for regressing the return of an individual security (the dependent variable) to the overall market return. The beta coefficient is the slope of the regression line. The beta for a security may also be calculated by dividing the covariance of the return on the market and the return on the security by the variance of the return on the market. Answer (B) is incorrect because beta equals the covariance of the returns on the market and on the security divided by the variance of the return on the market. Answer (C) is incorrect because beta equals the covariance of the returns on the market and on the security divided by the variance of the return on the market. Answer (D) is incorrect because beta equals the covariance of the returns on the market and on the security divided by the variance of the return on the market.

129 . Answer (B) is correct. Operating leverage is a measure of the degree to which fixed costs are used in the production process. A company with a higher percentage of fixed costs (higher operating leverage) has greater risk than one in the same industry that relies more heavily on variable costs. The DOL equals the percentage change in net operating income divided by the percentage change in sales. Thus, profits became more sensitive to changes in sales volume as the DOL increases. Answer (A) is incorrect because a firm with higher operating leverage has higher fixed costs and lower variable costs. Answer (C) is incorrect because a firm with higher leverage will be relatively more profitable than a firm with lower leverage when sales are high. The opposite is true when sales are low. Answer (D) is incorrect because a firm with higher leverage is more risky. Its reliance on fixed costs is greater.

RPCPA, AICPA. CMA & CIA EXAMINATION QUESTIONS Page 54 of 55

Page 55: T04 - Risks & Cost of Capital

MANAGEMENT ADVISORY SERVICES RISKS

142 . Answer (A) is correct. An insurance contract (a policy) must satisfy the usual requirements: offer and acceptance, consideration, legality, and capacity of the parties. The insured must have an insurable interest in the subject matter of the contract. Also, the subject matter generally must exist at the time of contracting. In the contract, the insurer makes a promise to pay a stated amount for loss or injury incurred as a result of a contingent event. Answer (B) is incorrect because the payee may be a stranger to the contract. A person who insures his/her own life names a third party as a beneficiary. Moreover, not every insurer is an insurance company, and the contingent event insured against must involve a risk. Answer (C) is incorrect because there is no general requirement that an insurance contract be written. Answer (D) is incorrect because the contract may often be oral, and, if the risk is not already in existence, the transaction is in essence a wager, not insurance.

Answer (A) is incorrect because life insurance policies often do not cover death while the insured is in the military. Answer (B) is incorrect because life insurance policies often do not cover death as a result of a noncommercial air flight. Answer (C) is incorrect because a lapsed policy may often be reinstated by payment of overdue premiums plus interest.

RPCPA, AICPA. CMA & CIA EXAMINATION QUESTIONS Page 55 of 55